Разное

Матрица математика как решать: теоремы и примеры нахождения определителей

Содержание

теоремы и примеры нахождения определителей

Содержание:

В общем случае правило вычисления определителей
$n$-го порядка
является довольно громоздким. Для определителей второго и третьего порядка существуют рациональные способы их вычислений.

Вычисления определителей второго порядка

Чтобы вычислить определитель матрицы второго порядка, надо от произведения
элементов главной диагонали отнять произведение
элементов побочной диагонали:

$$\left| \begin{array}{ll}{a_{11}} & {a_{12}} \\ {a_{21}} & {a_{22}}\end{array}\right|=a_{11} \cdot a_{22}-a_{12} \cdot a_{21}$$

Пример

Задание. Вычислить определитель второго порядка
$\left| \begin{array}{rr}{11} & {-2} \\ {7} & {5}\end{array}\right|$

Решение. $\left| \begin{array}{rr}{11} & {-2} \\ {7} & {5}\end{array}\right|=11 \cdot 5-(-2) \cdot 7=55+14=69$

Ответ. $\left| \begin{array}{rr}{11} & {-2} \\ {7} & {5}\end{array}\right|=69$

Методы вычисления определителей третьего порядка

Для вычисления определителей третьего порядка существует такие правила.

Правило треугольника

Схематически это правило можно изобразить следующим образом:

Произведение элементов в первом определителе, которые соединены прямыми,
берется со знаком «плюс»; аналогично, для второго определителя — соответствующие произведения берутся со знаком «минус», т.е.

$$\left| \begin{array}{ccc}{a_{11}} & {a_{12}} & {a_{13}} \\ {a_{21}} & {a_{22}} & {a_{23}} \\ {a_{31}} & {a_{32}} & {a_{33}}\end{array}\right|=a_{11} a_{22} a_{33}+a_{12} a_{23} a_{31}+a_{13} a_{21} a_{32}-$$

$$-a_{11} a_{23} a_{32}-a_{12} a_{21} a_{33}-a_{13} a_{22} a_{31}$$

Слишком сложно?

Методы вычисления определителей не по зубам? Тебе ответит эксперт через 10 минут!

Пример

Задание. Вычислить определитель $\left| \begin{array}{rrr}{3} & {3} & {-1} \\ {4} & {1} & {3} \\ {1} & {-2} & {-2}\end{array}\right|$ методом треугольников.

Решение. $\left| \begin{array}{rrr}{3} & {3} & {-1} \\ {4} & {1} & {3} \\ {1} & {-2} & {-2}\end{array}\right|=3 \cdot 1 \cdot(-2)+4 \cdot(-2) \cdot(-1)+$

$$+3 \cdot 3 \cdot 1-(-1) \cdot 1 \cdot 1-3 \cdot(-2) \cdot 3-4 \cdot 3 \cdot(-2)=54$$

Ответ. $\left| \begin{array}{rrr}{3} & {3} & {-1} \\ {4} & {1} & {3} \\ {1} & {-2} & {-2}\end{array}\right|=54$

Правило Саррюса

Справа от определителя дописывают первых два столбца и произведения элементов на главной диагонали и на диагоналях, ей
параллельных, берут со знаком «плюс»; а произведения элементов побочной диагонали и диагоналей, ей параллельных,
со знаком «минус»:

$$-a_{13} a_{22} a_{31}-a_{11} a_{23} a_{32}-a_{12} a_{21} a_{33}$$

Пример

Задание. Вычислить определитель $\left| \begin{array}{rrr}{3} & {3} & {-1} \\ {4} & {1} & {3} \\ {1} & {-2} & {-2}\end{array}\right|$ с помощью правила Саррюса.

Решение.

$$+(-1) \cdot 4 \cdot(-2)-(-1) \cdot 1 \cdot 1-3 \cdot 3 \cdot(-2)-3 \cdot 4 \cdot(-2)=54$$

Ответ. $\left| \begin{array}{rrr}{3} & {3} & {-1} \\ {4} & {1} & {3} \\ {1} & {-2} & {-2}\end{array}\right|=54$

Разложение определителя по строке или столбцу

Определитель равен сумме произведений элементов строки определителя на их
алгебраические дополнения. Обычно выбирают
ту строку/столбец, в которой/ом есть нули. Строку или столбец, по которой/ому ведется разложение, будет обозначать стрелкой.

Пример

Задание. Разложив по первой строке, вычислить определитель $\left| \begin{array}{lll}{1} & {2} & {3} \\ {4} & {5} & {6} \\ {7} & {8} & {9}\end{array}\right|$

Решение. $\left| \begin{array}{lll}{1} & {2} & {3} \\ {4} & {5} & {6} \\ {7} & {8} & {9}\end{array}\right| \leftarrow=a_{11} \cdot A_{11}+a_{12} \cdot A_{12}+a_{13} \cdot A_{13}=$

$1 \cdot(-1)^{1+1} \cdot \left| \begin{array}{cc}{5} & {6} \\ {8} & {9}\end{array}\right|+2 \cdot(-1)^{1+2} \cdot \left| \begin{array}{cc}{4} & {6} \\ {7} & {9}\end{array}\right|+3 \cdot(-1)^{1+3} \cdot \left| \begin{array}{cc}{4} & {5} \\ {7} & {8}\end{array}\right|=-3+12-9=0$

Ответ. $\left| \begin{array}{lll}{1} & {2} & {3} \\ {4} & {5} & {6} \\ {7} & {8} & {9}\end{array}\right|=0$

Этот метод позволяет вычисление определителя свести к вычислению определителя более низкого порядка.

Пример

Задание. Вычислить определитель $\left| \begin{array}{lll}{1} & {2} & {3} \\ {4} & {5} & {6} \\ {7} & {8} & {9}\end{array}\right|$

Решение. Выполним следующие
преобразования над строками определителя: из второй строки отнимем четыре
первых, а из третьей первую строку, умноженную на семь, в результате, согласно свойствам определителя, получим определитель,
равный данному.

$$\left| \begin{array}{ccc}{1} & {2} & {3} \\ {4} & {5} & {6} \\ {7} & {8} & {9}\end{array}\right|=\left| \begin{array}{ccc}{1} & {2} & {3} \\ {4-4 \cdot 1} & {5-4 \cdot 2} & {6-4 \cdot 3} \\ {7-7 \cdot 1} & {8-7 \cdot 2} & {9-7 \cdot 3}\end{array}\right|=$$

$$=\left| \begin{array}{rrr}{1} & {2} & {3} \\ {0} & {-3} & {-6} \\ {0} & {-6} & {-12}\end{array}\right|=\left| \begin{array}{ccc}{1} & {2} & {3} \\ {0} & {-3} & {-6} \\ {0} & {2 \cdot(-3)} & {2 \cdot(-6)}\end{array}\right|=0$$

Определитель равен нулю, так как вторая и третья строки являются пропорциональными.

Ответ. $\left| \begin{array}{lll}{1} & {2} & {3} \\ {4} & {5} & {6} \\ {7} & {8} & {9}\end{array}\right|=0$

Для вычисления определителей четвертого порядка и выше применяется либо разложение по строке/столбцу, либо приведение
к треугольному виду, либо с помощью теоремы Лапласа.

Разложение определителя по элементам строки или столбца

Пример

Задание. Вычислить определитель
$\left| \begin{array}{llll}{9} & {8} & {7} & {6} \\ {5} & {4} & {3} & {2} \\ {1} & {0} & {1} & {2} \\ {3} & {4} & {5} & {6}\end{array}\right|$ , разложив его по элементам какой-то строки или какого-то столбца.

Решение. Предварительно выполним
элементарные преобразования над строками определителя, сделав
как можно больше нулей либо в строке, либо в столбце. Для этого вначале от первой строки отнимем девять третьих,
от второй — пять третьих и от четвертой — три третьих строки, получаем:

$$\left| \begin{array}{cccc}{9} & {8} & {7} & {6} \\ {5} & {4} & {3} & {2} \\ {1} & {0} & {1} & {2} \\ {3} & {4} & {5} & {6}\end{array}\right|=\left| \begin{array}{cccc}{9-1} & {8-0} & {7-9} & {6-18} \\ {5-5} & {4-0} & {3-5} & {2-10} \\ {1} & {0} & {1} & {2} \\ {0} & {4} & {2} & {0}\end{array}\right|=\left| \begin{array}{rrrr}{0} & {8} & {-2} & {-12} \\ {0} & {4} & {-2} & {-8} \\ {1} & {0} & {1} & {2} \\ {0} & {4} & {2} & {0}\end{array}\right|$$

Полученный определитель разложим по элементам первого столбца:

$$\left| \begin{array}{rrrr}{0} & {8} & {-2} & {-12} \\ {0} & {4} & {-2} & {-8} \\ {1} & {0} & {1} & {2} \\ {0} & {4} & {2} & {0}\end{array}\right|=0+0+1 \cdot(-1)^{3+1} \cdot \left| \begin{array}{rrr}{8} & {-2} & {-12} \\ {4} & {-2} & {-8} \\ {4} & {2} & {0}\end{array}\right|+0$$

Полученный определитель третьего порядка также разложим по элементам строки и столбца, предварительно получив нули,
например, в первом столбце.{2+2} \cdot \left| \begin{array}{ll}{2} & {4} \\ {4} & {8}\end{array}\right|=$$

$$=4 \cdot(2 \cdot 8-4 \cdot 4)=0$$

Ответ. $\left| \begin{array}{cccc}{9} & {8} & {7} & {6} \\ {5} & {4} & {3} & {2} \\ {1} & {0} & {1} & {2} \\ {3} & {4} & {5} & {6}\end{array}\right|=0$

Замечание

Последний и предпоследний определители можно было бы и не вычислять,
а сразу сделать вывод о том, что они равны нулю, так как содержат пропорциональные строки.

Приведение определителя к треугольному виду

С помощью элементарных преобразований над строками или столбцами определитель приводится к треугольному виду и тогда его
значение, согласно свойствам определителя, равно произведению
элементов стоящих на главной диагонали.

Пример

Задание. Вычислить определитель
$\Delta=\left| \begin{array}{rrrr}{-2} & {1} & {3} & {2} \\ {3} & {0} & {-1} & {2} \\ {-5} & {2} & {3} & {0} \\ {4} & {-1} & {2} & {-3}\end{array}\right|$ приведением его к треугольному виду.

Решение. Сначала делаем нули в первом столбце под главной диагональю. Все преобразования
будет выполнять проще, если элемент $a_{11}$ будет
равен 1. Для этого мы поменяем местами первый и второй столбцы определителя, что, согласно свойствам определителя,
приведет к тому, что он сменит знак на противоположный:

$$\Delta=\left| \begin{array}{rrrr}{-2} & {1} & {3} & {2} \\ {3} & {0} & {-1} & {2} \\ {-5} & {2} & {3} & {0} \\ {4} & {-1} & {2} & {-3}\end{array}\right|=-\left| \begin{array}{rrrr}{1} & {-2} & {3} & {2} \\ {0} & {3} & {-1} & {2} \\ {2} & {-5} & {3} & {0} \\ {-1} & {4} & {2} & {-3}\end{array}\right|$$

Далее получим нули в первом столбце, кроме элемента $a_{11}$ ,
для этого из третьей строки вычтем две первых, а к четвертой строке прибавим первую, будем иметь:

$$\Delta=-\left| \begin{array}{rrrr}{1} & {-2} & {3} & {2} \\ {0} & {3} & {-1} & {2} \\ {0} & {-1} & {-3} & {-4} \\ {0} & {2} & {5} & {-1}\end{array}\right|$$

Далее получаем нули во втором столбце на месте элементов, стоящих под главной диагональю. И снова, если
диагональный элемент будет равен $\pm 1$ , то
вычисления будут более простыми. Для этого меняем местами вторую и третью строки (и при этом меняется на
противоположный знак определителя):

$$\Delta=\left| \begin{array}{rrrr}{1} & {-2} & {3} & {2} \\ {0} & {-1} & {-3} & {-4} \\ {0} & {3} & {-1} & {2} \\ {0} & {2} & {5} & {-1}\end{array}\right|$$

Далее делаем нули во втором столбце под главной диагональю, для этого поступаем следующим образом:
к третьей строке прибавляем три вторых, а к четвертой — две вторых строки, получаем:

$$\Delta=\left| \begin{array}{rrrr}{1} & {-2} & {3} & {2} \\ {0} & {-1} & {-3} & {-4} \\ {0} & {0} & {-10} & {-10} \\ {0} & {0} & {-1} & {-9}\end{array}\right|$$

Далее из третьей строки выносим (-10) за определитель и делаем нули в третьем столбце под
главной диагональю, а для этого к последней строке прибавляем третью:

$$\Delta=-10 \left| \begin{array}{rrrr}{1} & {-2} & {3} & {2} \\ {0} & {-1} & {-3} & {-4} \\ {0} & {0} & {1} & {1} \\ {0} & {0} & {-1} & {-9}\end{array}\right|=$$

$$=-10 \cdot \left| \begin{array}{cccc}{1} & {-2} & {3} & {2} \\ {0} & {-1} & {-3} & {-4} \\ {0} & {0} & {1} & {1} \\ {0} & {0} & {0} & {-8}\end{array}\right|=(-10) \cdot 1 \cdot(-1) \cdot 1 \cdot(-8)=-80$$

Ответ. $\Delta=-80$

Теорема Лапласа

Теорема

Пусть $\Delta$ — определитель
$n$-го порядка. Выберем в нем произвольные
$k$ строк (или столбцов), причем
$k \leq n-1$ . Тогда сумма произведений всех
миноров
$k$-го порядка, которые содержатся в выбранных
$k$ строках (столбцах), на их
алгебраические дополнения равна определителю.

Пример

Задание. Используя теорему Лапласа, вычислить определитель
$\left| \begin{array}{rrrrr}{2} & {3} & {0} & {4} & {5} \\ {0} & {1} & {0} & {-1} & {2} \\ {3} & {2} & {1} & {0} & {1} \\ {0} & {4} & {0} & {-5} & {0} \\ {1} & {1} & {2} & {-2} & {1}\end{array}\right|$

Решение. Выберем в данном определителе пятого порядка две строки —
вторую и третью, тогда получаем (слагаемые, которые равны нулю, опускаем):

$$\left| \begin{array}{rrrrr}{2} & {3} & {0} & {4} & {5} \\ {0} & {1} & {0} & {-1} & {2} \\ {3} & {2} & {1} & {0} & {1} \\ {0} & {4} & {0} & {-5} & {0} \\ {1} & {1} & {2} & {-2} & {1}\end{array}\right|=\left| \begin{array}{cc}{1} & {-1} \\ {4} & {-5}\end{array}\right| \cdot(-1)^{2+4+2+4} \cdot \left| \begin{array}{ccc}{2} & {0} & {5} \\ {3} & {1} & {1} \\ {1} & {2} & {1}\end{array}\right|+$$

$$+\left| \begin{array}{ll}{1} & {2} \\ {4} & {0}\end{array}\right| \cdot(-1)^{2+4+2+5} \cdot \left| \begin{array}{rrr}{2} & {0} & {4} \\ {3} & {1} & {0} \\ {1} & {2} & {-2}\end{array}\right|+\left| \begin{array}{cc}{-1} & {2} \\ {-5} & {0}\end{array}\right| \cdot(-1)^{2+4+5} \cdot \left| \begin{array}{ccc}{2} & {3} & {0} \\ {3} & {2} & {1} \\ {1} & {1} & {2}\end{array}\right|=$$

$$=-23+128+90=195$$

Ответ. $\left| \begin{array}{rrrrr}{2} & {3} & {0} & {4} & {5} \\ {0} & {1} & {0} & {-1} & {2} \\ {3} & {2} & {1} & {0} & {1} \\ {0} & {4} & {0} & {-5} & {0} \\ {1} & {1} & {2} & {-2} & {1}\end{array}\right|=195$

Читать дальше: обратная матрица.

определение, свойства и примеры решения задач

Задание. Вычислить $AB$ и $BA$,
если $ A=\left( \begin{array}{rr}{1} & {-1} \\ {2} & {0} \\ {3} & {0}\end{array}\right) $ ,
$ B=\left( \begin{array}{ll}{1} & {1} \\ {2} & {0}\end{array}\right) $

Решение. Так как $ A=A_{3 \times 2} $ , а $ B=B_{2 \times 2} $ ,
то произведение возможно и результатом операции умножения будет матрица $ C=C_{3 \times 2} $ , а это матрица вида
$ C=\left( \begin{array}{ll}{c_{11}} & {c_{12}} \\ {c_{21}} & {c_{22}} \\ {c_{31}} & {c_{32}}\end{array}\right) $ .

Вычислим
элементы матрицы $C$ :

$ c_{11}=a_{11} \cdot b_{11}+a_{12} \cdot b_{21}=1 \cdot 1+(-1) \cdot 2=-1 $

$ c_{12}=a_{11} \cdot b_{12}+a_{12} \cdot b_{22}=1 \cdot 1+(-1) \cdot 0=1 $

$ c_{21}=a_{21} \cdot b_{11}+a_{22} \cdot b_{21}=2 \cdot 1+0 \cdot 2=2 $

$ c_{22}=a_{21} \cdot b_{12}+a_{22} \cdot b_{22}=2 \cdot 1+0 \cdot 0=2 $

$ c_{31}=a_{31} \cdot b_{11}+a_{32} \cdot b_{21}=3 \cdot 1+0 \cdot 2=3 $

$ c_{32}=a_{31} \cdot b_{12}+a_{32} \cdot b_{22}=3 \cdot 1+0 \cdot 0=3 $

Итак, $ C=A B=\left( \begin{array}{rr}{-1} & {1} \\ {2} & {2} \\ {3} & {3}\end{array}\right) $ .

Выполним произведения в более компактном виде:

$ C=A B=\left( \begin{array}{rr}{1} & {-1} \\ {2} & {0} \\ {3} & {0}\end{array}\right)_{3 \times 2} \cdot \left( \begin{array}{ll}{1} & {1} \\ {2} & {0}\end{array}\right)_{2 \times 2}= $

$ =\left( \begin{array}{ccc}{1 \cdot 1+(-1) \cdot 2} & {1 \cdot 1+(-1) \cdot 0} \\ {2 \cdot 1+0 \cdot 2} & {2 \cdot 1+0 \cdot 0} \\ {3 \cdot 1+0 \cdot 2} & {3 \cdot 1+0 \cdot 0}\end{array}\right)=\left( \begin{array}{rr}{-1} & {1} \\ {2} & {2} \\ {3} & {3}\end{array}\right) $

Найдем теперь произведение $ D=B A=B_{2 \times 2} \cdot A_{3 \times 2} $. Так как
количество столбцов матрицы $B$ (первый сомножитель) не совпадает с
количеством строк матрицы $A$ (второй сомножитель), то данное произведение
неопределенно. Умножить матрицы в данном порядке невозможно.

Ответ. $ A B=\left( \begin{array}{rr}{-1} & {1} \\ {2} & {2} \\ {3} & {3}\end{array}\right) $ .
В обратном порядке умножить данные матрицы невозможно, так как количество столбцов матрицы
$B$ не совпадает с количеством строк матрицы $A$ .

Matrices & Linear Algebra | Mathematica & Wolfram Language for Math Students—Fast Intro

В Языке Wolfram матрицы представляются как списки списков:

In[1]:=

{{1, 2}, {3, 4}}

Их можно вводить в табличном виде, используя CTRL+ ENTER для добавления строк и CTRL+ , для добавления столбцов:

In[2]:=

{
 {a, b},
 {c, d}
}
Out[2]=

Функция MatrixForm позволяет отобразить матрицу в классическом виде:

In[3]:=

MatrixForm[{{a, b}, {c, d}}]
Out[3]=

Матрицы можно создавать с помощью итерационных функций:

In[1]:=

Table[x + y, {x, 1, 3}, {y, 0, 2}]
Out[1]=

Или импортировать данные, которые представляют собой матрицу:

In[2]:=

Import["data.csv"]
Out[2]=

IdentityMatrix, DiagonalMatrix и другие встроенные функции используются для создания матриц специального вида.

Стандартные матричные операции работают поэлементно:

In[1]:=

{1, 2, 3} {a, b, c}
Out[1]=

Вычисление произведения двух матриц:

In[2]:=

{{1, 2}, {3, 4}}.{{a, b}, {c, d}}
Out[2]=

Вычисление детерминанта:

In[3]:=

Det[{{a, b}, {c, d}}]
Out[3]=

Поиск обратной матрицы:

In[4]:=

Inverse[{{1, 1}, {0, 1}}]
Out[4]=

Функция LinearSolve используется для решения систем линейных уравнений:

In[1]:=

LinearSolve[{{1, 1}, {0, 1}}, {x, y}]
Out[1]=

Реализованы также функции для минимизации и декомпозиции матриц.2+1) dx с пределами интегрирования от минус бесконечности -∞ до плюс бесконечности +∞;

Для этого на странице https://www.kontrolnaya-rabota.ru/s/integral/nesobstvennyij/ в форму вводим данные, и получим подробный ответ(!):

>> тут <<

Двойной интеграл

В курсе высшей математики иногда требуется посчитать двойной интеграл, и вот — данный сайт решит указанный вами двойной интеграл.

К примеру, если вам требуется решить интеграл ∫ dx ∫x*sin(x*y) dy с пределами интегрирования от 0 до x и числа пи на два до числа пи.

Для этого на странице https://www.kontrolnaya-rabota.ru/s/integral/dvoinoi/ вводим данные, и получим очень подробный ответ:

>> где-то тут <<

Тройной интеграл

Тройной интеграл вы с легкостью решите из курса высшей математики.

Воспользуйтесь сервисом, находящимся по адресу https://www.kontrolnaya-rabota.ru/s/integral/troinoi/

Видео пример для двойного интеграла

Обратная матрица. Примеры вычисления

Нахождение обратной матрицы является важной составляющей в разделе линейной алгебры. С помощью таких матриц, если они существуют, можно быстро найти решение системы линейных уравнений.

Матрицаназывается обратной к матрице,если выполняются следующие равенства.

.

Если определитель матрицыотличен от нуля, то матрицу называют не особо или невырожденной.

Для того, чтобы матрица имела обратную необходимо и достаточно, чтобы она была невырожденной

Алгоритм нахождения обратной матрицы

Пусть имеем квадратную матрицу

и нужно найти обратную к ней. Для этого нужно выполнить следующие действия:

1. Найти определитель матрицы. Если он не равен нулю то выполняем следующие действия. В противном случае данная матрица вырождена и для нее не существует обратной

2. Найти алгебраические дополнения элементов матрицы . Они равны минорам, умноженным на в степени суммы строки и столбца, для которого ищем.

3. Составить матрицу из алгебраических дополнений элементов матрицы матрицы и протранспонировать ее. Эта матрица называется присоединенной или союзной и обозначается .

4. Разделить присоединенную матрицу на детерминант . Полученная матрица будет обратной и иметь свойства, которые изложены в начале статьи.

———————————————

Пример 1.

Найти матрицу, обратную к матрице (Дубовик В.П., Юрик И.И. «Высшая математика. Сборник задач»)

1) (1.127)

2) (1.130)

3) (1.133)

Решение.

1)Находим определитель матрицы

Так как детерминант не равен нулю (), то обратная матрица существует. Находим матрицу, составленную из алгебраических дополнений

Матрица дополнений примет вид

Транспонируем ее и получаем присоединенную

Разделим ее на определитель и получим обратную

Видим, что в случае, когда определитель равен единице присоединена и обратная матрицы совпадают.

2) Вычисляем определитель матрицы

Находим матрицу алгебраических дополнений

Конечный вид матрицы дополнений

Транспонируем ее и находим союзную матрицу

Находим обратную матрицу

3) Вычислим детерминант матрицы. Для этого разложим его на первую строчку. В результате получим два отличны от нуля слагаемые

Находим матрицу алгебраических дополнений. Расписание определителя проводим по строкам и столбцам, в которых больше нулевых элементов (обозначены черным цветом).

Конечный вид матрицы дополнений следующий

Транспонируем ее и находим присоединенную матрицу

Поскольку определитель матрицы равен единице то обратная матрица совпадает с присоединенной. Данный пример назад.

При вычислениях обратной матрицы типичными являются ошибки связанные с неправильными знаками при вычислении определителя и матрицы дополнений.

———————————————

——————————

Системы линейных уравнений (Лекция №14)

Системой m линейных уравнений с n неизвестными называется система вида

где aij и bi (i=1,…,m; b=1,…,n) – некоторые известные
числа, а x1,…,xn – неизвестные. В обозначении
коэффициентов aij первый индекс iобозначает номер уравнения,
а второй j – номер неизвестного, при
котором стоит этот коэффициент.

Коэффициенты при неизвестных будем записывать в виде матрицы , которую назовём матрицей
системы
.

Числа, стоящие в
правых частях уравнений, b1,…,bm называются свободными членами.

Совокупность n чисел c1,…,cn называется решением данной системы, если каждое уравнение
системы обращается в равенство после подстановки в него чисел c1,…,cn вместо соответствующих
неизвестных x1,…,xn.

Наша задача
будет заключаться в нахождении решений системы. При этом могут возникнуть три
ситуации:

  1. Система может иметь единственное решение.
  2. Система может иметь бесконечное множество решений. Например, . Решением этой системы является любая пара чисел, отличающихся
    знаком.
  3. И третий случай, когда система вообще не имеет решения. Например, , если бы решение существовало, то x1 + x2 равнялось бы одновременно нулю и единице.

Система линейных
уравнений, имеющая хотя бы одно решение, называется совместной. В противном случае, т.е. если система не имеет решений,
то она называется несовместной.

Рассмотрим
способы нахождения решений системы.

МАТРИЧНЫЙ МЕТОД РЕШЕНИЯ СИСТЕМ ЛИНЕЙНЫХ УРАВНЕНИЙ

Матрицы дают возможность
кратко записать систему линейных уравнений. Пусть дана система из 3-х уравнений
с тремя неизвестными:

Рассмотрим
матрицу системы и матрицы столбцы
неизвестных и свободных членов

Найдем произведение

т.е. в результате
произведения мы получаем левые части уравнений данной системы. Тогда пользуясь
определением равенства матриц данную систему можно записать в виде

или короче AX=B.

Здесь матрицы A и B известны, а матрица X неизвестна. Её и нужно
найти, т.к. её элементы являются решением данной системы. Это уравнение
называют матричным уравнением.

Пусть
определитель матрицы отличен от нуля |A| ≠ 0. Тогда матричное уравнение решается следующим образом.
Умножим обе части уравнения слева на матрицу A-1, обратную матрице A: . Поскольку A-1A = E и EX = X, то получаем решение
матричного уравнения в виде X = A-1B.

Заметим, что
поскольку обратную матрицу можно найти только для квадратных матриц, то
матричным методом можно решать только те системы, в которых число уравнений
совпадает с числом неизвестных
. Однако, матричная запись системы возможна и
в случае, когда число уравнений не равно числу неизвестных, тогда матрица A не будет квадратной и поэтому
нельзя найти решение системы в виде X = A-1B.

Примеры. Решить системы уравнений.

  1. Найдем матрицу
    обратную матрице A.

    ,

    Таким образом, x = 3, y = – 1.

  2. Итак, х1=4,х2=3,х3=5.

  3. Решите матричное уравнение: XA+B=C, где

    Выразим искомую
    матрицу X из заданного уравнения.

    Найдем матрицу А-1.

    Проверка:

  4. Решите матричное уравнение AX+B=C, где

    Из уравнения
    получаем .

    Следовательно,

ПРАВИЛО КРАМЕРА

Рассмотрим
систему 3-х линейных уравнений с тремя неизвестными:

Определитель третьего порядка, соответствующий матрице системы, т.е.
составленный из коэффициентов при неизвестных,

называется определителем системы.

Составим ещё три определителя следующим образом: заменим в определителе D последовательно
1, 2 и 3 столбцы столбцом свободных членов

Тогда можно доказать следующий результат.

Теорема (правило Крамера). Если определитель системы Δ
≠ 0, то рассматриваемая система имеет одно и только одно решение, причём

Доказательство. Итак,
рассмотрим систему 3-х уравнений с тремя неизвестными. Умножим 1-ое уравнение
системы на алгебраическое дополнение A11 элемента a11, 2-ое уравнение – на A21
и 3-е – на A31:

Сложим эти уравнения:

Рассмотрим каждую из скобок и правую часть этого уравнения. По теореме о
разложении определителя по элементам 1-го столбца

.

Далее рассмотрим коэффициенты при x2:

Аналогично можно показать, что и .

Наконец несложно заметить, что

Таким образом, получаем равенство: .

Следовательно, .

Аналогично выводятся равенства и , откуда и следует утверждение теоремы.

Таким образом, заметим, что если определитель системы Δ
≠ 0, то система имеет единственное решение и обратно. Если же
определитель системы равен нулю, то система либо имеет бесконечное множество
решений, либо не имеет решений, т.е. несовместна.

Примеры. Решить систему
уравнений

  1. Итак, х=1, у=2, z=3.

  2. Решите систему уравнений
    при различных значениях параметра p:

    Система имеет единственное решение, если Δ
    ≠ 0.

    . Поэтому .

    1. При
    2. При p = 30 получаем систему уравнений которая не имеет решений.
    3. При p = –30 система принимает вид и, следовательно,
      имеет бесконечное множество решений x=y, yÎR.

МЕТОД ГАУССА

Ранее рассмотренные методы можно применять при решении только тех систем,
в которых число уравнений совпадает с числом неизвестных, причём определитель
системы должен быть отличен от нуля. Метод Гаусса является более универсальным
и пригоден для систем с любым числом уравнений. Он заключается в
последовательном исключении неизвестных из уравнений системы.

Вновь рассмотрим систему из трёх уравнений с тремя неизвестными:

.

Первое уравнение оставим без изменения, а из 2-го и 3-го исключим
слагаемые, содержащие x1. Для этого второе
уравнение разделим на а21 и умножим на –а11, а
затем сложим с 1-ым уравнением. Аналогично третье уравнение разделим на а31 и умножим на –а11, а затем сложим с первым. В результате
исходная система примет вид:

Теперь из последнего уравнения исключим слагаемое, содержащее x2. Для этого третье уравнение разделим на , умножим на и сложим со вторым.
Тогда будем иметь систему уравнений:

Отсюда из последнего уравнения легко найти x3,
затем из 2-го уравнения x2 и, наконец, из 1-го – x1.

При использовании метода Гаусса уравнения при необходимости можно менять
местами.

Часто вместо того, чтобы писать новую систему уравнений, ограничиваются
тем, что выписывают расширенную матрицу системы:

и затем
приводят её к треугольному или диагональному виду с помощью элементарных преобразований.

К элементарным преобразованиям
матрицы относятся следующие преобразования:

  1. перестановка строк или столбцов;
  2. умножение строки на число, отличное от нуля;
  3. прибавление к одной строке другие строки.

Примеры: Решить системы
уравнений методом Гаусса.

  1. Вернувшись к системе уравнений, будем иметь

  2. Выпишем расширенную матрицу системы и сведем ее к треугольному виду.

    Вернувшись к системе уравнений, несложно заметить, что третье уравнения
    системы будет ложным, а значит, система решений не имеет.

  3. Разделим вторую строку матрицы на 2 и поменяем местами первый и третий
    столбики. Тогда первый столбец будет соответствовать коэффициентам при
    неизвестной z, а третий – при x.

    Вернемся к системе уравнений.

    Из третьего уравнения выразим одну неизвестную через другую и подставим в
    первое.

Таким образом, система имеет бесконечное множество решений.

Калькулятор матриц — простой, удобный и совместим со старыми смартфонами

Приближается новый учебный год, и многие студенты уже начали к нему готовиться, несмотря на солнечную погоду, которая настраивает скорее на развлечения, чем на серьезные занятия. Среди того, что нужно современному студенту, — не только талант к математике, учебники, тетради и ручки, но и набор программных инструментов, который поможет в решении даже самых сложных задач. Одним из таких приложений является Калькулятор матриц от разработчика Koliuzhnov Viacheslav, которое можно загрузить из Google Play.

Приложение не только позволяет выполнять над матрицами различные действия, но и шаг за шагом последовательно отображает процесс выполняемых вычислений. Матрицы в рассматриваемом калькуляторе можно не только складывать и вычитать, но также умножать и возводить в степень. Матрица может быть также умножена на определенное число.

Кроме того, как следует из описания рассматриваемого приложения, с его помощью может быть найден детерминант матрицы или осуществлено ее транспортирование. Данный калькулятор предназначен в том числе и для решения систем линейных алгебраических уравнений (СЛАУ) методами Крамера и Гаусса.

Калькулятор дает возможность задать значения матриц (A, B) и указать их размерность (по умолчанию 3×3). Матрица C — результат произведенных вычислений. В приложении, воспользовавшись специальными экранными кнопками, можно поменять матрицы местами (к примеру, [A⇿B]).

В настройках приложения (Настройки) могут быть заданы точность вычислений, размер шрифта математических формул и возможность не отключать экран в процессе работы приложения.

Умножение матрицы в данном приложении может осуществляться как на вещественное число, так и на простую дробь.

В дизайне приложения преобладает светло-серый цвет, удобный для зрительного восприятия и психологически настраивающий на то, что приложение предназначено для решения сложных математических задач и пользователю необходимо сосредоточиться на правильном вводе значений, не отвлекаясь на красочные элементы интерфейса. Страницы вычислений стилизованы под привычную студенческую тетрадь в клетку.

Отсутствие эстетических излишеств в значительной мере упрощает использование приложения, делая его интерфейс понятным даже для тех пользователей, которые не считают высшую математику своей сильной стороной. Такой дизайнерский подход также позволяет осуществлять столь сложные вычисления даже на телефонах, которые помогают своим пользователям уже не первый год.

Приложением смогут воспользоваться обладатели девайсов под управлением операционных систем Android начиная с версии 2.3.3. Таким образом, рассматриваемый «Калькулятор матриц» доступен даже тем пользователям операционной системы Android, которые не считают нужным часто менять смартфон.

В Google Play присутствует также простой калькулятор интегралов и дифференциальных уравнений. Обладатели Android-устройств могут одним касанием решать квадратные уравнения и системы линейных уравнений. Рассмотренное приложение в очередной раз показывает, что смартфон — не только замена игровой консоли, но и мощный инструмент, помогающий достигать успехов в учебе.

Стало ли студентам с появлением Android-девайсов проще изучать точные науки?

Приложение: Калькулятор матриц
Разработчик: Koliuzhnov Viacheslav
Категория: Образование
Версия: 1.5
Цена: Бесплатно
Скачать: Google Play

Приложением уже заинтересовались: 262 человека

Решение систем линейных уравнений с использованием матриц

Привет! Эта страница будет иметь смысл только тогда, когда вы немного знаете о системах линейных уравнений и матриц, поэтому, пожалуйста, пойдите и узнайте о них, если вы их еще не знаете!

Пример

Одним из последних примеров систем линейных уравнений был этот:

Пример: Решить

  • х + у + г = 6
  • 2y + 5z = −4
  • 2x + 5y — z = 27

Затем мы решили его, используя метод «исключения»… но мы можем решить это с помощью Матриц!

Использование матриц упрощает жизнь, потому что мы можем использовать компьютерную программу (например, Матричный калькулятор), чтобы выполнять всю «обработку чисел».

Но сначала нам нужно написать вопрос в матричной форме.

в матричной форме?

ОК. Матрица — это массив чисел, верно?

Матрица

Ну, подумайте об уравнениях:

х + л + z = 6
2 года + 5z = −4
2x + 5лет z = 27

Их можно было бы превратить в таблицу чисел вот так:

1 1 1 = 6
0 2 5 = −4
2 5 -1 = 27

Мы могли бы даже разделить числа до и после «=» на:

1 1 1 6
0 2 5 и −4
2 5 -1 27

Теперь похоже, что у нас есть 2 матрицы.

На самом деле у нас есть третий, это [x y z]:

Почему [x y z] идет туда? Потому что, когда мы умножаем матрицы, левая часть становится:

Это исходная левая часть приведенных выше уравнений (вы можете это проверить).

Матричное решение

Мы можем написать это:

как это:

AX = B

где

  • A — это матрица 3×3 коэффициентов x, y и z
  • X — это x, y и z, и
  • .

  • B — это 6, −4 и 27

Тогда (как показано на странице инверсии матрицы) решение таково:

X = A -1 B

Что это значит?

Это означает, что мы можем найти значения x, y и z (матрица X), умножив , инверсную матрицу A , на матрицу B .

Итак, давайте продолжим и сделаем это.

Во-первых, нам нужно найти , обратную матрице A (при условии, что она существует!)

Используя Матричный калькулятор, получаем:

(для упрощения чисел я оставил определитель 1 / вне матрицы)

Затем умножьте A -1 на B (мы снова можем использовать Матричный калькулятор):

И готово! Решение:

x = 5,
y = 3,
z = −2

Как и на странице Системы линейных уравнений.

Довольно изящный и элегантный, человек думает, а компьютер производит вычисления.

Просто для удовольствия … Сделай это снова!

Для удовольствия (и для того, чтобы помочь вам учиться), давайте проделаем все это снова, но сначала поставим матрицу «X».

Я хочу показать вам этот путь, потому что многие люди думают, что вышеприведенное решение настолько изящно, что это, должно быть, единственный способ.

Так что решим так:

XA = B

И из-за способа умножения матриц нам нужно настроить матрицы по-другому.Строки и столбцы необходимо поменять местами («транспонировать»):

И XA = B выглядит так:

Матричное решение

Тогда (также показано на странице инверсии матрицы) решение следующее:

X = BA -1

Это то, что мы получаем для A -1 :

На самом деле он похож на инверсию, которую мы получали раньше, но транспонированную (строки и столбцы меняются местами).

Затем умножаем B на A -1 :

И решение то же:

x = 5, y = 3 и z = −2

Это выглядело не так красиво, как предыдущее решение, но оно показывает нам, что существует более одного способа составления и решения матричных уравнений.Только будьте осторожны со строками и столбцами!

Как работать с матрицами (алгебра 2, матрицы) — Mathplanet

Матрицы считаются равными, если они имеют одинаковые размеры и если каждый элемент одной матрицы равен соответствующему элементу другой матрицы. Вы можете умножить матрицу на любую константу, это называется скалярным умножением.


Пример

$$ 2 \ begin {bmatrix} 1 & 2 \\ 3 & 4 \ end {bmatrix} = \ begin {bmatrix} {\ color {green} 2} \ cdot 1 & {\ color {green} 2} \ cdot2 \ \ {\ color {green} 2} \ cdot3 & {\ color {green} 2} \ cdot4 \ end {bmatrix} = \ begin {bmatrix} 2 & 4 \\ 6 & 8 \ end {bmatrix} $$

Матрицы одинакового размера могут быть добавлены и вычтены, а матрицы совместимых размеров могут быть умножены.


Пример

Сложите две матрицы A и B:

$$ A = \ begin {bmatrix} {\ color {green} 2} & {\ color {green} -1} \\ {\ color {green} 1} & {\ color {green} 0} \ end { bmatrix} B = \ begin {bmatrix} {\ color {blue} 1} & {\ color {blue} 4} \\ {\ color {blue} 2} & \, {\ color {blue} 3} \ end { bmatrix} $$

Это возможно, поскольку A и B, поскольку обе матрицы имеют две строки и два столбца. Мы добавляем каждый элемент в матрицу A к соответствующему элементу в матрице B:

$$ A + B = \ begin {bmatrix} {\ color {green} 2} + {\ color {blue} 1} & {\ color {green} -1} + {\ color {blue} 4} \\ {\ color {green} 1} + {\ color {blue} 2} & {\ color {green} 0} + \, {\ color {blue} 3} \ end {bmatrix} = \ begin {bmatrix} 3 & 3 \\ 3 & 3 \ end {bmatrix} $$

Матрицы вычитаются таким же образом.

Если вы умножите матрицу A p * q и матрицу B m * n , произведение будет

$$ A_ {p \ times q} + B_ {m \ times n} = (AB) _ {p \ times n} $$

Элемент в g-й строке и h-м столбце AB является суммой произведения соответствующих элементов в g-й строке A и h-м столбце B. Вы можете перемножить только две матрицы. если количество столбцов в первой матрице и количество строк во второй равны.


Пример

Умножаем матрицы A и B:

$$ A = \ begin {bmatrix} {\ color {green} 1} & {\ color {green} 3} \\ {\ color {green} -1} & {\ color {green} 0} \ end { bmatrix} B = \ begin {bmatrix} {\ color {blue} 2} & {\ color {blue} 1} & {\ color {blue} 1} \\ {\ color {blue} -1} & {\ color {blue} 2} & {\ color {blue} 4} \ end {bmatrix} $$

Это возможно, поскольку первая матрица содержит 2 столбца, а вторая — 2 строки.

$$ \\ AB = \ begin {bmatrix} {\ color {green} 1} \ cdot {\ color {blue} 2} + {\ color {green} 3} \ cdot {\ color {blue} -1} & {\ color {зеленый} 1} \ cdot {\ color {blue} 1} + {\ color {green} 3} \ cdot {\ color {blue} 2} & {\ color {green} 1} \ cdot { \ color {blue} 1} + {\ color {green} 3} \ cdot {\ color {blue} 4} \\ {\ color {green} -1} \ cdot {\ color {blue} 2} + {\ цвет {зеленый} 0} \ cdot {\ color {синий} -1} & {\ color {зеленый} -1} \ cdot {\ color {blue} 1} + {\ color {зеленый} 0} \ cdot {\ color {blue} 2} & {\ color {green} -1} \ cdot {\ color {blue} 1} + {\ color {green} 0} \ cdot {\ color {blue} 4} \ end {bmatrix} = \\ \\ = \ begin {bmatrix} -1 & 7 & 13 \\ -2 & -1 & -1 \ end {bmatrix} $$


Видеоурок

Если возможно, выполните указанную операцию:

$$ \ begin {bmatrix} 2 & 0 \\ 1 & 3 \\ -4 & 1 \ end {bmatrix} \ cdot \ begin {bmatrix} 6 & 2 & 3 \\ 0 & -2 & 4 \ end {bmatrix} $$

Решение матричных уравнений

А

матричное уравнение

уравнение, в котором переменная обозначает

матрица

.

Вы можете решить более простые матричные уравнения, используя

матрица сложения

и

скалярное умножение

.


Примеры 1:

Решить для матрицы

Икс

:

Икс

+

[

3

2

1

0

]

знак равно

[

6

3

7

1

]

Икс

+

[

3

2

1

0

]

[

3

2

1

0

]

знак равно

[

6

3

7

1

]

[

3

2

1

0

]

Икс

+

[

0

0

0

0

]

знак равно

[

6

3

3

2

7

1

1

0

]

Икс

знак равно

[

3

1

6

1

]


Примеры 2:

Решить для матрицы

Икс

:

Икс

[

9

3

6

0

]

знак равно

[

4

0

12

10

]

Икс

[

9

3

6

0

]

знак равно

[

4

0

12

10

]

Икс

[

9

3

6

0

]

+

[

9

3

6

0

]

знак равно

[

4

0

12

10

]

+

[

9

3

6

0

]

Икс

[

0

0

0

0

]

знак равно

[

4

+

(

9

)

0

+

(

3

)

12

+

6

10

+

0

]

Икс

знак равно

[

5

3

18

10

]


Решение систем линейных уравнений с использованием матриц:

Матричные уравнения можно использовать для

решать системы линейных уравнений

используя левую и правую части уравнений.


Примеры 3:

Решите систему уравнений с помощью матриц:

{

7

Икс

+

5

y

знак равно

3

3

Икс

2

y

знак равно

22

7

Икс

+

5

y

знак равно

3

3

Икс

2

y

знак равно

22

[

7

Икс

+

5

y

3

Икс

2

y

]

знак равно

[

3

22

]

Запишите матрицу слева как произведение коэффициентов и переменных.

[

7

5

3

2

]

[

Икс

y

]

знак равно

[

3

22

]

коэффициент

Переменная

постоянный

матрица

матрица

матрица

Сначала найдите обратную матрицу коэффициентов.Обратное

[

7

5

3

2

]

является

1

7

(

2

)

(

3

)

(

5

)

[

2

5

3

7

]

знак равно

1

29

[

2

5

3

7

]

знак равно

[

2

29

5

29

3

29

7

29

]

Затем умножьте каждую сторону матричного уравнения на

обратная матрица

.Поскольку матричное умножение

нет

коммутативной, обратная матрица должна быть слева на

каждый

сторона матричного уравнения.

[

2

29

5

29

3

29

7

29

]

[

7

5

3

2

]

[

Икс

y

]

знак равно

[

2

29

5

29

3

29

7

29

]

[

3

22

]

[

1

0

0

1

]

[

Икс

y

]

знак равно

[

4

5

]

В

единичная матрица

слева подтверждает, что обратная матрица была рассчитана правильно.

[

Икс

y

]

знак равно

[

4

5

]

Решение

(

4

,

5

)

.

Использование матриц для решения систем уравнений

Матричные уравнения

Матрицы

могут использоваться для компактного написания и работы с системами множественных линейных уравнений.

Цели обучения

Определить, как матрицы могут представлять систему уравнений

Основные выводы

Ключевые моменты
  • Если [latex] A [/ latex] является матрицей [latex] m \ times n [/ latex], а [latex] x [/ latex] обозначает вектор-столбец (например, [latex] n \ times 1 [/ latex] матрица) [латекс] n [/ latex] переменных [latex] x_1, x_2,…, x_n [/ latex], и [latex] b [/ latex] представляет собой [латекс] m \ times 1 [/ latex ] Вектор-столбец, то матричное уравнение имеет вид: [latex] Ax = b [/ latex].
Ключевые термины
  • матрица : прямоугольное расположение чисел или членов, имеющее различное применение, например, преобразование координат в геометрии, решение систем линейных уравнений в линейной алгебре и представление графиков в теории графов.

Матрицы можно использовать для компактного написания и работы с системами уравнений. Как мы узнали в предыдущих разделах, матрицами можно манипулировать так же, как и нормальным уравнением. Это очень полезно, когда мы начинаем работать с системами уравнений.Полезно понять, как организовать матрицы для решения этих систем.

Написание системы уравнений с матрицами

Можно решить эту систему, используя метод исключения или замены, но это также возможно сделать с помощью матричной операции. Прежде чем приступить к настройке матриц, важно сделать следующее:

  • Убедитесь, что все уравнения написаны одинаково, то есть переменные должны быть в одном порядке.
  • Убедитесь, что одна часть уравнения — это только переменные и их коэффициенты, а другая сторона — просто константы.

Решение системы линейных уравнений с использованием обратной матрицы требует определения двух новых матриц: [latex] X [/ latex] — это матрица, представляющая переменные системы, а [latex] B [/ latex] — матрица, представляющая константы. Используя матричное умножение, мы можем определить систему уравнений с таким же количеством уравнений в качестве переменных, как:

[латекс] \ displaystyle A \ cdot X = B [/ латекс]

Чтобы решить систему линейных уравнений с использованием обратной матрицы, пусть [latex] A [/ latex] будет матрицей коэффициентов, пусть [latex] X [/ latex] будет переменной матрицей, и пусть [latex] B [/ latex ] — постоянная матрица.

Учитывая систему:

[латекс] \ displaystyle \ begin {align} x + 8y & = 7 \\ 2x-8y & = — 3 \ end {align} [/ latex]

Матрица коэффициентов:

[латекс] A = \ begin {bmatrix} 1 & 8 \\ 2 & -8 \ end {bmatrix} [/ latex]

Матрица переменных:

[латекс] \ displaystyle X = \ begin {bmatrix} x \\ y \ end {bmatrix} [/ latex]

Постоянная матрица:

[латекс] \ displaystyle B = \ begin {bmatrix} 7 \\ -3 \ end {bmatrix} [/ latex]

Таким образом, чтобы решить систему [latex] AX = B [/ latex], для [latex] X [/ latex] умножьте обе стороны на обратную величину [latex] A [/ latex], и мы получим решение:

[латекс] \ Displaystyle X = (A ^ {- 1}) B [/ латекс]

Если существует обратный [латекс] \ left (A ^ {- 1} \ right) [/ latex], эта формула решит систему.

Если матрица коэффициентов необратима, система может быть несовместимой и не иметь решения, или быть зависимой и иметь бесконечно много решений.

Матрицы и операции со строками

Две матрицы эквивалентны строкам, если одна может быть заменена другой последовательностью элементарных операций со строками.

Цели обучения

Объясните, как использовать операции со строками и почему они создают эквивалентные матрицы

Основные выводы

Ключевые моменты
  • Элементарная операция со строкой — это любое из следующих действий: переключение строк (перестановка двух строк матрицы), умножение строк (умножение строки матрицы на ненулевую константу) или сложение строк (добавление к одной строке матрицы до некоторого числа, кратного другой строке).
  • Если строки матрицы представляют собой систему линейных уравнений, то пространство строк состоит из всех линейных уравнений, которые могут быть выведены алгебраически из уравнений системы.
Ключевые термины
  • пространство строки : набор всех возможных линейных комбинаций его векторов-строк.
  • эквивалент строки : В линейной алгебре, когда одна матрица может быть заменена другой последовательностью элементарных операций со строкой.

Элементарные операции со строками (ERO)

В линейной алгебре две матрицы эквивалентны строкам, если одна может быть заменена другой последовательностью элементарных операций со строками.В качестве альтернативы, две матрицы [latex] m \ times n [/ latex] эквивалентны строкам тогда и только тогда, когда они имеют одинаковое пространство строк. Пространство строки матрицы представляет собой набор всех возможных линейных комбинаций ее векторов-строк. Если строки матрицы представляют собой систему линейных уравнений, то пространство строк состоит из всех линейных уравнений, которые могут быть выведены алгебраически из уравнений системы. Две матрицы одинакового размера эквивалентны строкам тогда и только тогда, когда соответствующие однородные системы имеют одинаковый набор решений или, что эквивалентно, матрицы имеют одно и то же нулевое пространство.Поскольку элементарные операции со строками обратимы, эквивалентность строк является отношением эквивалентности. Обычно обозначается тильдой (~).

Операция элементарной строки — это любой из следующих трех ходов:

  1. Переключение строк (перестановка): поменять местами две строки матрицы.
  2. Умножение строк (масштаб): умножение строки матрицы на ненулевую константу.
  3. Сложение строк (сводная): прибавить к одной строке матрицы несколько значений, кратных другой строке.

Создание эквивалентных матриц с использованием элементарных операций со строками

Поскольку матрица по существу является коэффициентами и константами линейной системы, три операции со строками сохраняют матрицу.Например, замена двух строк просто означает изменение их положения в матрице. Кроме того, при решении системы линейных уравнений методом исключения, умножение строк будет таким же, как умножение всего уравнения на число для получения аддитивных обратных величин, так что переменная сокращается. Наконец, добавление строк аналогично методу исключения, когда для получения переменной выбирается сложение или вычитание одинаковых членов уравнений. Следовательно, операции со строками сохраняют матрицу и могут использоваться как альтернативный метод для решения системы уравнений.

Пример 1: Покажите, что эти две матрицы эквивалентны строкам:

[латекс] \ displaystyle A = \ begin {pmatrix} 1 & -1 & 0 \\ 2 & 1 & 1 \ end {pmatrix} \ quad B = \ begin {pmatrix} 3 & 0 & 1 \\ 0 & 3 & 1 \ end {pmatrix} [/ латекс]

Начните с [latex] A [/ latex], добавьте вторую строку к первой:

[латекс] \ displaystyle A = \ begin {pmatrix} 3 & 0 & 1 \\ 2 & 1 & 1 \ end {pmatrix} [/ latex]

Затем умножьте вторую строку на 3 и вычтите первую строку из второй:

[латекс] \ displaystyle A = \ begin {pmatrix} 3 & 0 & 1 \\ 3 & 3 & 2 \ end {pmatrix} [/ latex]

Наконец, вычтите первую строку из второй:

[латекс] \ displaystyle A = \ begin {pmatrix} 3 & 0 & 1 \\ 0 & 3 & 1 \ end {pmatrix} [/ latex]

Вы можете видеть, что [latex] A = B [/ latex], что мы достигли с помощью серии элементарных операций со строками.

Сокращение строк: решение системы линейных уравнений

В редукторе рядов, линейная система:

[латекс] \ displaystyle x + 3y-2z = 5 \\ 3x + 5y + 6z = 7 \ 2x + 4y + 3z = 8 [/ latex]

Представлен в виде расширенной матрицы:

[латекс] \ displaystyle A = \ begin {pmatrix} 1 & 3 & -2 & 5 \\ 3 & 5 & 6 & 7 \\ 2 & 4 & 3 & 8 \ end {pmatrix} [/ latex]

Затем эта матрица модифицируется с использованием операций с элементарными строками до тех пор, пока она не достигнет уменьшенной формы эшелона строк.

Поскольку эти операции обратимы, полученная расширенная матрица всегда представляет собой линейную систему, эквивалентную исходной.

Существует несколько специальных алгоритмов сокращения строк расширенной матрицы, простейшими из которых являются исключение Гаусса и исключение Гаусса-Жордана. Это вычисление может быть выполнено вручную (с использованием трех типов ERO) или на калькуляторе с использованием матричной функции «rref» (сокращенная форма эшелона строк).

Окончательная матрица представлена ​​в виде уменьшенного ряда строк и представляет систему [латекс] x = -15 [/ latex], [latex] y = 8 [/ latex] [latex] z = 2 [/ latex].

[латекс] \ displaystyle A = \ begin {pmatrix} 1 & 0 & 0 & 0 & -15 \\ 0 & 1 & 0 & 8 \\ 0 & 0 & 1 & 2 \ end {pmatrix} [/ latex]

Упрощение матриц с помощью операций со строками

Используя элементарные операции, метод исключения Гаусса приводит матрицы к форме эшелона строк.

Цели обучения

Используйте операции с элементарными строками, чтобы представить матрицу в упрощенном виде

Основные выводы

Ключевые моменты
  • Поскольку элементарные операции со строками сохраняют пространство строк матрицы, пространство строк формы эшелона строк такое же, как и у исходной матрицы.
  • Существует три типа операций с элементарными строками: меняют местами две строки, умножают строку на ненулевой скаляр и добавляют к одной строке скалярное значение, кратное другой.
  • На практике обычно не рассматривают системы в терминах уравнений, а вместо этого используют расширенную матрицу (которая также подходит для компьютерных манипуляций).
Ключевые термины
  • Расширенная матрица : Матрица, полученная путем добавления столбцов двух заданных матриц, обычно с целью выполнения одних и тех же элементарных операций со строками для каждой из данных матриц.

С помощью конечной последовательности элементарных операций со строками, называемых исключением по Гауссу, любую матрицу можно преобразовать в форму эшелона строк. Это преобразование необходимо для решения системы линейных уравнений.

Прежде чем углубляться в детали, следует упомянуть несколько ключевых терминов:

  • Расширенная матрица : расширенная матрица — это матрица, полученная путем добавления столбцов двух заданных матриц, обычно с целью выполнения одних и тех же операций с элементарной строкой для каждой из данных матриц.
  • Форма верхнего треугольника : Квадратная матрица называется верхней треугольной, если все элементы ниже главной диагонали равны нулю. Треугольная матрица — это нижняя или верхняя треугольная матрица. Матрица, имеющая одновременно верхний и нижний треугольники, является диагональной матрицей.
  • Элементарные операции со строками : Поменять местами строки, добавить строки или умножить строки.

Исключение по Гауссу

  1. Напишите расширенную матрицу для линейных уравнений.
  2. Используйте элементарные операции со строками в расширенной матрице [latex] [A | b] [/ latex], чтобы преобразовать [latex] A [/ latex] в форму верхнего треугольника. Если на диагонали находится ноль, переключайте строки, пока на его месте не окажется ненулевое значение.
  3. Используйте обратную замену, чтобы найти решение.

Пример 1: Решите систему методом исключения Гаусса:

[латекс] \ displaystyle 2x + y-z = 8 \\ -3x-y + 2z = -11 \ -2x + y + 2z = -3 [/ latex]

Запишите расширенную матрицу:

[латекс] \ left [\ begin {array} {rrr | r} 2 & 1 & -1 & 8 \\ -3 & -1 & 2 & -11 \\ -2 & 1 & 2 & -3 \ end {array} \ right] [/ latex]

Используйте элементарные операции со строками, чтобы уменьшить матрицу до уменьшенной формы эшелона строк:

[латекс] \ left [\ begin {array} {rrr | r} 1 & 0 & 0 & 2 \\ 0 & 1 & 0 & 3 \\ 0 & 0 & 1 & -1 \ end {array} \ right ] [/ латекс]

Используя элементарные операции со строками для получения сокращенной формы эшелона строк (‘rref’ в калькуляторе), решение системы отображается в последнем столбце: [latex] x = 2, y = 3, z = -1 [/ latex] .

Решение матричных уравнений за один этап с помощью резистивных массивов точек пересечения

Значение

Линейная алгебра используется практически во всех научных и инженерных дисциплинах, например, в физике, статистике, машинном обучении и обработке сигналов. Решение матричных уравнений, таких как линейная система или уравнение с собственным вектором, выполняется путем факторизации матриц или итерационного умножения матриц на обычных компьютерах, что требует больших вычислительных ресурсов. Вычисления в оперативной памяти с аналоговой резистивной памятью продемонстрировали высокую эффективность использования времени и энергии за счет реализации умножения матрицы на вектор за один шаг по закону Ома и закону Кирхгофа.Однако решение матричных уравнений за одну операцию остается открытой проблемой. Здесь мы показываем, что схема обратной связи с перекрестной резистивной памятью может решать алгебраические задачи, такие как системы линейных уравнений, собственные векторы матриц и дифференциальные уравнения, всего за один шаг.

Abstract

Обычные цифровые компьютеры могут выполнять расширенные операции с помощью последовательности элементарных логических функций из 2 или более битов. В результате сложные задачи, такие как решение линейной системы или решение дифференциального уравнения, требуют большого количества вычислительных шагов и широкого использования модулей памяти для хранения отдельных битов.Для ускорения выполнения таких сложных задач вычисления в памяти с резистивной памятью представляют собой многообещающее направление благодаря хранению аналоговых данных и физическим вычислениям в памяти. Здесь мы показываем, что массив точек пересечения резистивных запоминающих устройств может напрямую решать систему линейных уравнений или находить собственные векторы матрицы. Эти операции выполняются всего за один шаг, благодаря физическим вычислениям по законам Ома и Кирхгофа, а также благодаря подключению с отрицательной обратной связью в схеме коммутации.Алгебраические задачи демонстрируются на оборудовании и применяются к классическим вычислительным задачам, таким как ранжирование веб-страниц и решение уравнения Шредингера за один шаг.

Задачи линейной алгебры, такие как решение систем линейных уравнений и вычисление собственных векторов матриц, лежат в основе современных научных вычислений и задач, требующих обработки большого количества данных. Традиционно эти проблемы в форме матричных уравнений решаются матричными факторизациями или итеративным матричным умножением (1, 2), которые являются дорогостоящими в вычислительном отношении с полиномиальной временной сложностью, например.г., O ( N 3 ), где N — размер проблемы. Поскольку традиционные компьютеры все чаще сталкиваются с ограничениями масштабирования технологии комплементарного металл-оксид-полупроводник (КМОП) (3), а также из-за затрат энергии и задержек при перемещении данных между памятью и вычислительными блоками (4), улучшение вычислений производительность с увеличением аппаратных ресурсов становится сложной и неэкономичной. Чтобы обойти эти фундаментальные ограничения, вычисления в памяти недавно стали многообещающим методом для проведения вычислений на месте, т.е.е., в блоке памяти (5). Одним из примеров являются вычисления в массивах точек пересечения, которые могут ускорить умножение матрицы на вектор (MVM) по закону Ома и закону Кирхгофа с аналоговой и реконфигурируемой резистивной памятью (5⇓⇓ – 8). MVM в памяти был адаптирован для нескольких задач, включая сжатие изображений (5), разреженное кодирование (6) и обучение глубоких нейронных сетей (7, 8). Однако решение матричных уравнений, таких как линейная система Ax = b , за одну операцию остается открытой проблемой.Здесь мы показываем, что схема обратной связи, включающая реконфигурируемую резистивную решетку в точках пересечения, может обеспечить решение алгебраических задач, таких как системы линейных уравнений, собственные векторы матрицы и дифференциальные уравнения, всего за один шаг.

Резистивная память — это двухполюсные элементы, которые могут изменять свою проводимость в ответ на приложенное напряжение (9, 10). Благодаря своему энергонезависимому и реконфигурируемому поведению резистивные запоминающие устройства широко исследовались и разрабатывались для запоминающих устройств (11, 12), логики с отслеживанием состояния (13⇓ – 15), вычислений в памяти (5, 6, 16, 17), и нейроморфные вычислительные приложения (7, 8, 18, 19).Резистивная память включает в себя различные концепции устройств, такие как резистивная коммутационная память (RRAM, ссылки 9–12), память с изменением фазы (PCM, ссылка 20) и магнитная память с передачей вращения по крутящему моменту (21). Реализованные в архитектуре массива точек пересечения, резистивная память может естественным образом ускорить операции с большими объемами данных с повышенной эффективностью времени / энергии по сравнению с классическими цифровыми вычислениями (5, 6, 17). Также недавно было показано, что итерированные операции MVM с резистивными массивами точек пересечения могут решать системы линейных уравнений в сочетании с цифровыми компьютерами с плавающей запятой (22).Чем выше желаемая точность решения, тем больше итераций требуется для завершения операции. Однако итерация поднимает фундаментальный предел для достижения высокой вычислительной производительности с точки зрения энергии и задержки.

Результаты

Схемы пересечения для решения системы линейных уравнений.

Рис. 1 A показывает предложенную схему обратной связи для решения системы линейных уравнений за один шаг, а аппаратная схема на печатной плате показана в приложении SI , рис.S1. Схема представляет собой матрицу узлов RRAM, каждое из которых состоит из пакета металл-изолятор-металл со слоем HfO 2 между верхним электродом из Ti и нижним электродом из C (15). Устройства показывают установленный переход от высокого сопротивления к низкому сопротивлению, когда положительное напряжение выше порогового значения V set применяется к Ti-электроду, и переход сброса от низкого сопротивления к высокому сопротивлению, когда отрицательное напряжение выше порога V сброс применяется к Ti-электроду.Многоуровневая работа также возможна путем выполнения заданного перехода при переменном максимальном (согласованном) токе I C или выполнении перехода в сброс при переменном максимальном напряжении В stop (23), как показано в приложении SI , Рис. S2. Массив точек пересечения 3 × 3 на рисунке может выполнять MVM с разомкнутым контуром, то есть путем приложения вектора напряжения В, к столбцам и измерения вектора тока I в строках без соединений строка-столбец, разрешенных с помощью операционные усилители (ОУ), которые показаны в приложении СИ , рис.S3. Измеренные токи дают скалярное произведение I = A · V между приложенными аналоговыми напряжениями и матрицей A значений проводимости RRAM в матрице точек пересечения. Результаты свидетельствуют о небольшой ошибке, обычно менее 8%, в основном из-за нелинейности проводимости в резистивных устройствах с перекрестными точками. Это соответствует предыдущим результатам, в которых точность MVM оказалась удовлетворительной (5), хотя и не соответствовала полностью цифровым операциям с одинарной и двойной точностью.

Рис. 1.

Решение систем линейных уравнений с массивом точек пересечения резистивных устройств. ( A ) Схема пересечения для решения линейной системы или инвертирования положительной матрицы. Элементы RRAM (красные цилиндры) расположены в точках пересечения между строками (синие полосы) и столбцами (зеленые полосы). ( Вставка , Справа ) Экспериментальные значения проводимости, отображающие элементы матрицы A . Единицы преобразования между матрицами / векторами с действительным знаком и физическими реализациями были: G 0 = 100 мкс, V 0 = 1 В и I 0 = 100 мкА для проводимости RRAM, входное / выходное напряжение и выходной / входной ток соответственно.Другие случаи также следуют этому соглашению, если не указано иное. ( B ) Схемы для вычисления скалярного произведения I = G · V по закону Ома и для вычисления скалярного деления V = — I / G с помощью TIA. ( C ) Измеренное решение линейной системы с вектором входного тока I = [0,2; 1; 1] I 0 . Экспериментальные выходные напряжения дают решение, очень близкое к аналитическому.( D ) Измеренное решение для линейных систем, а именно выходное напряжение, как функция параметра β , управляющего входным током, задаваемым I = β · [0,2; 1; 1] I 0 с −1 ≤ β ≤ 1. Экспериментальные решения (цветные кружки) сравниваются с аналитическими решениями (цветные линии) системы, что подтверждает точность физического расчета. ( E ) Обратная экспериментальная матрица A −1 , а именно измеренные выходные напряжения в трех последующих экспериментах с входным током I = [1; 0; 0] I 0 , [0; 1; 0] I 0 и [0; 0; 1] I 0 соответственно.Также показано аналитическое решение. ( Вставка ) Матричное произведение AA -1 очень близко к единичной матрице U , таким образом поддерживая экспериментальную инверсию.

Работа MVM является следствием физического закона Ома I = G · В , где G — проводимость устройства, В — приложенное напряжение, а I — измеренный ток ( Рис.1 B , Верх ).С другой стороны, обратная операция V = — I / G может быть получена для заданных I и G , просто нагнетая ток I в заземленном узле резистивного устройства. и измерение потенциала В, во втором узле. Это физическое разделение выполняется трансимпедансным усилителем (TIA) на рис. 1 B ( Bottom ), где ток вводится в узел инвертирующего входа OA, а проводимость обратной связи G соединяет вход и выходные узлы ОУ.Дифференциальное входное напряжение В + В на ОУ минимизировано высоким коэффициентом усиления ОУ, тем самым устанавливая виртуальную землю ( В, = 0) на инвертирующем входе. node (24, 25) и включение физического разделения. Это обеспечивает основу для схемы на рис. 1 A , которая решает систему линейных уравнений, выраженную матричной формулой: Ax = b, [1] где A — невырожденная квадратная матрица, отображаемая с поперечными значениями проводимости. -точечные устройства RRAM, b — известный вектор, а x — неизвестный вектор.В этой схеме входные токи I = — b подаются на ряды точек пересечения, подключенные к узлам виртуальной земли OA. В результате токи вынуждены автоматически распределяться между резистивными элементами в массиве точек пересечения, чтобы установить выходной потенциал В , удовлетворяющий A · V + I = 0, [2] что подразумевает В = — A -1 · I = x . Схема, аналогичная показанной на рис. 1 A ранее была представлена ​​в отчете International Roadmap for Devices and Systems (25) и предложена исх.26, хотя не было продемонстрировано возможности решения линейной системы с помощью экспериментов или моделирования.

Чтобы продемонстрировать концепцию на рис. 1 A , мы измерили выходные напряжения в матрице точек пересечения RRAM 3 × 3 на рис. 1 A , где также показана матрица проводимости. Все матрицы, принятые в экспериментах в этой работе, представлены в приложении SI, таблица S1. Вектор тока [ I 10 ; I 20 ; I 30 ] с I 10 = 20 мкА, I 20 = 100 мкА, и I 30 = 100 мкА, был применен к строкам массива, и результирующий потенциал в столбцах массива, т.е.е., [ V 10 ; V 20 ; V 30 ], измеряли, как показано на фиг. 1 C . Хорошее согласие (с относительными ошибками в пределах 3%) с аналитическим решением поддерживает функциональность цепи обратной связи, показанной на рис. 1 A для решения матричного уравнения в уравнении. 1 . Схема была дополнительно продемонстрирована путем линейного изменения входных токов в соответствии с I i = β I i 0 , где i = 1, 2 или 3, а β было изменяется равномерно в диапазоне от -1 до 1.Результаты представлены на рис. 1 D , где показаны измеренные выходные напряжения в сравнении с аналитическими решениями x = A -1 b . Ошибка остается ниже 10% для | β | > 0,5 ( SI Приложение , рис. S4). Примечательно, что уравнение. 1 физически решается всего за один шаг благодаря физическому MVM в массиве точек пересечения и соединению обратной связи, вынуждающему виртуальное заземление в рядах точек пересечения.

Та же концепция может быть расширена для вычисления инверсии матрицы A , удовлетворяющей AA -1 = U , где U — единичная матрица.Столбец i -й столбца A -1 может быть измерен как выходное напряжение, когда столбец i -й столбец U применяется в качестве входа, таким образом реализуя инверсию матрицы с шагом N . На рис. 1 E показаны измеренные элементы A −1 в сравнении с аналитически решенными элементами обратной матрицы, а относительные ошибки вычислены в приложении SI , рис. S5. Рис. 1 E ( вставка ) показывает, что экспериментальный продукт AA -1 хорошо аппроксимирует U , что дополнительно поддерживает вычисленную инверсию матрицы.

Схема на рис. 1 A — это, по сути, оператор инверсии матрицы, который можно использовать для решения линейных систем и инверсий матриц, в то время как массив точек пересечения без обратной связи является оператором матрицы, который, естественно, может использоваться для выполнить MVM. Поскольку схема инверсии матрицы является системой с отрицательной обратной связью, стабильность выходного напряжения требует, чтобы коэффициент усиления контура ( G контур ) каждого контура обратной связи был отрицательным (27). Анализ показывает, что условие G loop <0 выполняется, когда все знаки диагональных элементов A −1 положительны ( SI Приложение , рис.S6). Следуя этому руководству, была решена система линейных уравнений и инверсия матрицы 5 × 5, при этом матрица была реализована в виде массива дискретных резисторов в точках пересечения. Небольшая относительная погрешность около нескольких процентов в этом идеальном случае с дискретными резисторами свидетельствует о том, что высокая точность может быть достигнута с помощью точных и линейных устройств резистивной памяти ( SI Приложение , Рис. S7).

Решение линейной системы с положительными и отрицательными коэффициентами.

Поскольку в резистивном элементе проводимость может быть только положительной, схема на рис.1 может решать только линейные системы с положительной матрицей коэффициентов. Для решения линейных систем с неположительными коэффициентами должна быть принята схема со смешанной матрицей, показанная на рисунке 2. Здесь матрица A разделена на два массива точек пересечения в соответствии с A = B — C, где B и C положительны. На рис.2 A показана реализация массива с двумя точками пересечения, где входной ток I разделен схемой на два компонента I B и I C = I I B , передаваемый в ряды виртуальной земли B и C , соответственно.Аналоговые инверторы позволяют инвертировать напряжение между столбцами B и C . Исходя из закона Ома и закона тока Кирхгофа, выходное напряжение В OA определяется как B · V + C (−V) + I = 0, [3] или A · V + I = 0, который решает линейную систему уравнения. 1 с I = — b .

Рис. 2.

Обращение смешанной матрицы. ( A ) Схема двух массивов точек пересечения для инверсии матриц, где два массива точек пересечения содержат элементы матриц B ( Bottom ) и C ( Top ) с A = B C .Напряжение в матрице C инвертируется в другой с помощью аналоговых инверторов, в то время как входной ток вводится в линии виртуальной земли и разделяется на две матрицы. ( B ) Измеренные значения матриц A , B и C , при этом A = B C . В эксперименте матрица B была реализована в виде массива точек пересечения RRAM, а матрица C была реализована в виде массива точек пересечения дискретных резисторов.( C ) Измеренные значения обратной матрицы A -1 как функция аналитически рассчитанных элементов A -1 . Поскольку A −1 является положительной матрицей, ее можно инвертировать с помощью единственного массива точек пересечения, как показано на рисунке 1. ( D ) Значения проводимости для матрицы A −1 , реализованные в элементах RRAM , как функция экспериментальных значений A −1 в C .Чтобы устройства работали в области высокой проводимости, матрица A -1 была реализована с G 0 = 500 мкс для проводимости RRAM. ( E ) Измеренные элементы матрицы ( A −1 ) −1 как функция аналитических расчетов. I 0 = 500 мкА и В 0 = 1 В использовались для входного тока и выходного напряжения соответственно. ( F ) Измеренные элементы матрицы ( A −1 ) −1 как функция с исходной матрицей A , демонстрируя замечательную точность, несмотря на накопленные ошибки по двум последовательным процессам инверсии и устройству -процесс программирования.

Мы экспериментально продемонстрировали инверсию смешанной матрицы 3 × 3 A с двумя матрицами B и C , реализованными в массиве RRAM и массиве резисторов, соответственно. Значения A , B и C показаны на рис. 2 B , а на рис. 2 C показаны измеренные элементы A −1 как функция аналитического результаты, демонстрирующие хорошую точность. Чтобы дополнительно поддержать инверсию физической матрицы, мы инвертировали A -1 , которая является положительной матрицей, с одним массивом точек пересечения.Для этой цели элементы A -1 были сначала отображены как значения проводимости в массиве RRAM с использованием алгоритма программирования и проверки с ошибкой менее 5% (приложение SI , рис. S8). Хотя алгоритм программирования и проверки применялся к отдельному устройству RRAM за раз, массив точек пересечения подходит для параллельного программирования, чтобы значительно сократить время инициализации массива (28, 29). На рис. 2 D показаны измеренные значения проводимости RRAM как функция целевых значений, полученных из экспериментального A -1 на рис.2 С . Инверсия A −1 , то есть ( A −1 ) −1 , была вычислена схемой инверсии матриц, показанной на рис. 1 A , что дало результаты на рис. E . Вычисленное ( A −1 ) −1 сравнивается с исходной матрицей A на рис.2 F , которая поддерживает хорошую точность двойных инверсий ( A −1 ) -1 = А .Относительные ошибки вышеуказанных операций указаны в приложении SI , рис. S9.

Подобно схеме с одиночной матрицей точек пересечения на рис. 1 A , условие отрицательной обратной связи применяется к смешанной матрице A . Кроме того, поскольку матрица точек пересечения B непосредственно участвует в обратной связи с обратной связью с OA, матрица B также должна удовлетворять условию G loop <0. В качестве предложения для практических приложений. , эталонная матрица B , удовлетворяющая условию G loop , может быть принята в схеме со смешанной матрицей, в то время как матрица C может быть свободно размещена с помощью массива точек пересечения RRAM с условием C = B A .Чтобы продемонстрировать общность этой концепции, одномерное стационарное уравнение Фурье для диффузии тепла было решено с помощью схемы с перекрестными точками ( SI Приложение , рис. S10 и S11). Используя метод конечных разностей, дифференциальное уравнение сначала преобразуется в систему линейных уравнений, где характеристическая матрица A является смешанной трехдиагональной матрицей. Входные токи соответствуют известному термину, а именно рассеиваемой мощности в одномерной структуре.Решение дает профиль температуры вдоль эталонной структуры, которая решает численное уравнение Фурье.

Ключевым параметром для описания устойчивости решения линейной системы является число обусловленности κ матрицы (30). Число обусловленности отражает стабильность решения x при небольших изменениях известного члена b в уравнении. 1 , где чувствительность к возмущениям возрастает с увеличением числа обусловленности. Чтобы изучить влияние числа обусловленности на решение линейных систем в массивах резистивной памяти, мы смоделировали схемное обращение трех матриц 10 × 10 с увеличением числа обусловленности.Чтобы проверить стабильность решения, случайное изменение 0,1 или -0,1 было добавлено к каждому элементу в члене b уравнения Ax = b , где b — это i -й столбец единичная матрица U , x — это i -й столбец A -1 , а i был прогнут от 1 до 10 для вычисления всей обратной матрицы. Результаты представлены в приложении SI , рис. S12, что указывает на то, что ошибка вычисления увеличивается с увеличением числа обусловленности матрицы.

Влияние числа обусловленности также было проверено в экспериментах путем выполнения двойного обращения матрицы с большим числом обусловленности ( κ = 16,9) по сравнению с матрицей с κ = 9,5 на рис. 2. Номера условий для всех матриц в эксперименте сведены в SI Приложение , Таблица S1. Как показано в приложении SI , рис. S13, матрица с большим значением κ успешно инвертируется дважды, хотя ошибки вычислений больше, чем в случае на рис.2 ( SI Приложение , рис. S14). Следует отметить, что рассматриваемые в данной работе матрицы хорошо подготовлены. Для плохо обусловленной матрицы с чрезвычайно высоким числом обусловленности должны потребоваться дополнительные схемы, возможно, включая алгоритмы итеративного уточнения, которые могут поддерживаться обычным цифровым компьютером (22) или реализованы в массиве резистивной памяти (26). Ошибка, вызванная тепловым шумом и дробовым шумом компонентов в схеме пересечения, также увеличивается с увеличением числа условий, хотя представляет гораздо менее значительную проблему ( SI Приложение , рис.S15).

Схемы коммутации для вычисления собственных векторов.

Решение линейной системы в уравнении. 1 может быть дополнительно расширен до вычисления собственных векторов посредством физических вычислений в массиве точек пересечения. Уравнение для собственного вектора имеет вид Ax = λx, [4] где A — вещественная квадратная матрица, λ — ее собственное значение, а x — соответствующий собственный вектор. Рис. 3 A показывает схему собственного вектора, состоящую из цепи самоуправляемой обратной связи, где вектор напряжения В , сформированный в столбцах точек пересечения, формирует вектор тока I = A · В , с проводимость матрицы точек пересечения, отображающей матрицу A .Выходные токи преобразуются в напряжения с помощью TIA с резисторами обратной связи G λ , отображающими известное собственное значение λ . Затем выходные сигналы TIA инвертируются и возвращаются в столбцы точек пересечения. Комбинируя закон Ома и закон Кирхгофа, получаем — A · V / G λ = — V , следовательно, A · V = G λ 905 V , который удовлетворяет уравнению. 4 . Поскольку физические напряжения и токи могут иметь только действительные значения, схема собственных векторов применяется только к действительным собственным значениям и собственным векторам. Для положительной матрицы, согласно теореме Перрона – Фробениуса (31), наивысшее собственное значение должно быть положительным действительным числом, а его собственный вектор также состоит из положительных действительных чисел. В результате собственный вектор наивысшего собственного значения положительной матрицы всегда может быть решен с помощью перекрестной схемы. Если собственный вектор самого низкого отрицательного собственного значения действительный, его также можно измерить, удалив аналоговые инверторы в цепи обратной связи ( SI Приложение , рис.S16 A ). Обратите внимание, что схема собственного вектора на рис. 3 A работает автономно, подобно генератору с положительной обратной связью, благодаря активным TIA, устанавливающим вектор напряжения V .

Рис. 3.

Расчеты собственного вектора и PageRank. ( A ) Схема пересечения для решения уравнения собственных векторов Ax = λx , где x — собственный вектор, а λ — наивысшее положительное собственное значение положительной матрицы. вставка.Чтобы предотвратить нарушение проводимости RRAM при установке / сбросе, выходные напряжения OA были ограничены до ± 0,2 В. ( B ) Измеренные собственные векторы, соответствующие самому высокому положительному собственному значению и самому низкому отрицательному собственному значению, как функция нормированных собственных векторов полученные аналитическими решениями. Наивысшее положительное собственное значение и наименьшее отрицательное собственное значение были сохранены как проводимость обратной связи G λ TIA с проводимостью 940 и 331 мкс, соответственно.( C ) Система из четырех веб-страниц с соответствующими ссылками. Стрелка, указывающая со страницы i на страницу j , указывает на ссылку j на странице i , поэтому важность веб-страницы можно определить по количеству стрелок, указывающих на эту страницу. ( D ) Матрица ссылок для системы в C . Сумма элементов в каждом столбце равна 1, а все диагональные элементы равны нулю, поскольку страницы не ссылаются на себя. Единица преобразования была G 0 = 684 мкс для проводимости RRAM, чтобы минимизировать нелинейность RRAM.Наивысшее положительное собственное значение равно 1, что соответствует резисторам обратной связи с проводимостью G 0 . ( E ) Измеренный собственный вектор, представляющий оценки важности четырех страниц, как функция аналитически решенного нормализованного собственного вектора.

Схема собственного вектора на рис. 3 A была экспериментально продемонстрирована для массива точек пересечения RRAM со значениями проводимости G , отображающими матрицу A (рис. 3 A , вставка ) путем вычисления собственные векторы для наивысшего положительного собственного значения ( λ + = 9.41) и наименьшее отрицательное собственное значение ( λ = −3,31). На рис. 3 B показаны измеренные значения собственных векторов как функции нормированных собственных векторов, полученных с помощью аналитических решений. Пропорциональность между экспериментальными и рассчитанными собственными векторами на рисунке указывает на правильное физическое вычисление собственных векторов.

Хотя ограничение решения самыми высокими / самыми низкими собственными значениями может показаться неудобным, оказывается, что для многих приложений используются только самые высокие положительные или самые низкие отрицательные собственные значения.Например, в алгоритме PageRank (32, 33), который дает оценки важности веб-страниц для их ранжирования, собственный вектор матрицы ссылок вычисляется для наивысшего положительного собственного значения. Последний всегда равен 1, поскольку матрица связей является стохастической матрицей (33). На рис. 3 C показан пример четырех страниц с соответствующими ссылками, а на рис. 3 D показана соответствующая матрица ссылок, которая была реализована как значения проводимости массива точек пересечения RRAM 4 × 4.Используя схему собственного вектора на рис. 3 A , был решен собственный вектор матрицы ссылок для вычисления оценок важности страниц. Рис. 3 E показывает экспериментальные оценки по сравнению с аналитическими оценками, демонстрируя хорошую точность физического вычисления собственного вектора. Реальный случай PageRank описан в приложении SI , рис. S17.

Анализ схемы собственных векторов на рис.3 A показывает, что G петля в идеале должна быть равна 1 ( SI Приложение , Рис.S18), что, однако, никогда не может быть полностью удовлетворено в практических схемах. На практике G λ можно экспериментально выбрать так, чтобы G петля была немного больше 1, что позволяет правильно решить собственный вектор с приемлемой ошибкой. Фактически, хотя выход изначально увеличивается из-за цикла G > 1, нелинейность схемы, возникающая из-за насыщения выхода TIA, уменьшает цикл G до 1.С другой стороны, установка G loop меньше 1 приводит к нулевому выходному напряжению, чего, таким образом, следует избегать. Аналогично Рис. 2 A , решение для собственных векторов может быть расширено до смешанной матрицы A с помощью техники разделения с двумя массивами точек пересечения, соединенными аналоговыми инверторами ( SI Приложение , Рис. S16 B ).

Мы проверили физическое вычисление собственных векторов для решения одномерного не зависящего от времени уравнения Шредингера: HΨ = EΨ, [5] где H — оператор Гамильтона, E — собственное значение энергии, а Ψ — соответствующая собственная функция.Уравнение 5 может быть численно решена методом конечных разностей, давая задачу на собственный вектор, заданную уравнением. 4 , где A — трехдиагональная матрица коэффициентов, x — вектор значений в дискретных позициях, а λ — наивысшее / наименьшее собственное значение. Уравнение Шредингера было решено для квадратной потенциальной ямы, показанной на рис. 4 A , которая была разделена поровну на 32 сегмента ( SI Приложение , рис. S19 и S20).На рис. 4 B показана трехдиагональная смешанная матрица A 33 × 33, описывающая уравнения собственных векторов. Матрица A, разделена на две положительные трехдиагональные матрицы B и C , которые отображаются в значениях проводимости двух массивов точек пересечения, соответственно. Собственный вектор был рассчитан для основного состояния с энергией E = -4,929 эВ, что соответствует наименьшему отрицательному собственному значению задачи. Собственные значения и собственные векторы, полученные путем численного решения на цифровом компьютере, также указаны в приложении SI , рис.S19. На рис. 4 C показан собственный вектор, полученный с помощью схемы смоделированного собственного вектора, в сравнении с аналитически вычисленным собственным вектором. Физически вычисленная волновая функция хорошо согласуется с численным решением, которое дополнительно поддерживает физические вычисления в схемах пересечения точек для реальных приложений.

Рис. 4.

Решение уравнения Шредингера в схеме пересечения. ( A ) Прямоугольная яма потенциала V ( x ), принятая в уравнении Шредингера.Потенциальная яма имеет глубину -5 эВ и ширину 2 нм, в то время как решение проводится с общей шириной 3,2 нм, дискретизированной в 32 равных интервалах. ( B ) Матрица A размером 33 × 33, полученная в результате пространственной дискретизации уравнения Шредингера, и две положительные матрицы B и C , реализованные в массивах точек пересечения, с A = В С . Единица преобразования 100 мкс для 7,6195 эВ была принята в матрицах B и C .Две матрицы проводимости имеют одну и ту же цветовую полосу. Собственное значение в основном состоянии составляет -4,929 эВ, что отображается на проводимость (65 мкСм) резисторов обратной связи TIA. ( C ) Дискретная собственная функция основного состояния, полученная как смоделированное выходное напряжение в схеме пересечения по сравнению с аналитическими решениями. Обратите внимание, что пиковое напряжение составляет около 1,5 В напряжения питания ОУ из-за насыщения.

Обсуждение

Массивы точек пересечения позволяют решать широкий круг задач алгебры, от линейных систем до задач на собственные векторы, тем самым обеспечивая физическое решение дифференциальных уравнений, описывающих реальные проблемы в промышленности, экономике и здравоохранении.Решение основано на чрезвычайно простых схемных элементах, таких как имеющиеся в продаже OA и современные резистивные запоминающие устройства, такие как RRAM и PCM. Для сравнения, предыдущие решения линейных систем с использованием подхода квантовых вычислений (34, 35) менее привлекательны, поскольку квантовые схемы обычно работают при криогенных температурах и требуют специального оборудования и некоммерческих технологий. Другие предлагаемые решения с архитектурой нейронных сетей (36) или аналоговыми ускорителями на основе КМОП (37) основаны на итерационных операциях, что приводит к полиномиальному времени вычислений и стоимости.Напротив, массив точек пересечения позволяет быстро решить всего за один шаг без итераций. Время вычислений ограничено временем установления ОУ, которое может достигать нескольких наносекунд в передовой КМОП-технологии (38).

Чтобы оправдать ожидания практических приложений, схему коммутации следует масштабировать, чтобы продемонстрировать выполнимость схемы. Чтобы продемонстрировать масштабируемость схемы пересечения, решение системы линейных уравнений для матрицы коэффициентов модели 100 × 100 при моделировании показано в приложении SI , рис.S21. Результаты показывают, что линейная система точно решается схемой, которая поддерживает пригодность схемы коммутации для решения реальных проблем. Поскольку матричные коэффициенты хранятся в реальных наноразмерных устройствах с присущими им стохастическими вариациями, схема пересечения обеспечивает только приблизительное решение линейной задачи. Чтобы оценить влияние вариаций устройства, мы включили случайное отклонение проводимости каждого перекрестного устройства для матрицы 100 × 100 и рассчитали относительные ошибки выходных напряжений ( SI Приложение , рис.S22). Результаты моделирования показывают относительно низкие ошибки (около 10%) даже с отклонением в 10%. Таким образом, высокоточное хранение значений проводимости с помощью методов программирования и проверки имеет важное значение для повышения точности решения в зависимости от конкретных приложений. Нелинейная проводимость в резистивном элементе, физически возникающая из-за прыжковой проводимости и локального джоулева нагрева, также влияет на точность решения. Линейность проводимости может быть максимизирована путем увеличения проводимости устройства (5), что, однако, приводит к более высокому требованию энергии для перенастройки и работы схемы пересечения.Развитие технологии резистивной памяти, направленной на более высокую точность многоуровневого размещения и лучшую линейность проводимости, может улучшить схему пересечения для вычислений линейной алгебры в памяти.

По мере увеличения масштаба схемы коммутации паразитное сопротивление из-за плотной разводки межсоединений в массиве памяти может стать дополнительной проблемой. Чтобы оценить влияние паразитного сопротивления, мы смоделировали ту же линейную систему 100 × 100 из SI Приложение , рис.S21 с дополнительным паразитным сопротивлением провода ( SI Приложение , рис. S23). Для справки параметры межсоединений были взяты из Международной дорожной карты технологий для полупроводников на 65- и 22-нм технологических узлах (39). Относительные ошибки находятся в пределах ∼10 и 30% для узлов 65 и 22 нм соответственно. Эти результаты предполагают, что существует компромисс между масштабированием и точностью схемотехнических решений задач алгебры. Также следует отметить, что ошибки вычислений по существу продиктованы соотношением сопротивлений между сопротивлением устройства и паразитным сопротивлением.В результате точность вычислений может быть улучшена за счет увеличения сопротивления запоминающих устройств, что, в свою очередь, может вызвать проблему нелинейности проводимости, которая также влияет на точность вычислений. Мы пришли к выводу, что существует сложный компромисс между масштабированием, паразитным сопротивлением и нелинейностью устройства для оптимизации операций (40, 41). В этом сценарии трехмерная интеграция памяти точки пересечения, где плотность не обязательно приводит к увеличению сопротивления межсоединений, может повысить устойчивость точности вычислений к паразитному сопротивлению (42).

Хотя отсутствие итераций является очень привлекательной особенностью для быстрых вычислений, время, необходимое для программирования индивидуальных матричных коэффициентов в памяти, также следует учитывать для всесторонней оценки технологии. Хотя время записи в наших устройствах было относительно большим с целью точной настройки значений проводимости (см., Например, приложение SI, приложение , рис. S8), время программирования в реальном приложении могло бы быть значительно ускорено благодаря параллельному программирование (28, 29), схемы аналогового программирования (43), помимо субнаносекундной коммутации устройств RRAM (44) и устройств PCM (45).Кроме того, согласно концепции вычислений в памяти, одни и те же данные могут часто повторно использоваться для вычислений (42), таким образом, время программирования может играть незначительную роль в общем времени вычислений.

Хотя точность нашей схемы нельзя сравнивать с точностью решения с плавающей запятой в высокоточном цифровом компьютере, важно отметить, что требуемая точность может быть не высокой для всех приложений. На самом деле существует много случаев, когда задача линейной алгебры должна быть решена за короткое время, с низким бюджетом энергии и с достаточной устойчивостью к ошибкам.Например, в алгоритмах машинного обучения коэффициенты классификации / распознавания могут рассчитываться с некоторой погрешностью. Сетевые коэффициенты могут быть получены с помощью псевдообратной матрицы (46), вычисление которой может быть ускорено нашим подходом. Другим примером является ранжирование веб-страниц, где вычисленные оценки веб-сайта должны отображаться в правильном порядке, хотя некоторые неточности все же могут допускаться для отдельных оценок. Для аналогичных типов приложений наши схемы могут предоставить решение с отличным компромиссом между точностью, скоростью и потреблением энергии.

В заключение были представлены решения задач линейной алгебры в резистивных массивах точек пересечения. Такие задачи, как системы линейных уравнений, собственные векторы матриц и дифференциальные уравнения, решаются ( i ) за один шаг (и инверсия матрицы за N шагов), ( ii ) in situ в массиве памяти точек пересечения, и ( iii ) через физические законы, такие как закон Ома, закон Кирхгофа и механизмы обратной связи в схемах с обратной связью. Предлагаемые вычисления в памяти прокладывают путь для будущих приблизительных вычислительных систем в памяти для решения практических задач с большими данными с огромной экономией времени и энергии для широкого спектра реальных приложений.

Методы

Подробная информация о производстве и характеристиках устройства, проектировании схем и методах измерения приведена в Приложении SI .

Благодарности

Эта работа получила финансирование от Европейского исследовательского совета в рамках программы исследований и инноваций Европейского Союза Horizon 2020 (Соглашение о гранте 648635). Эта работа была частично выполнена на Polifab, предприятии по микро- и нанотехнологиям Миланского политехнического университета.

Сноски

  • Автор: З.С., Г.П., Д.И. спланированное исследование; Z.S., G.P., E.A., A.B., W.W. и D.I. проведенное исследование; З.С., Г.П., Д.И. проанализированные данные; и З.С. и Д. написал газету.

  • Авторы заявляют об отсутствии конфликта интересов.

  • Эта статья представляет собой прямое представление PNAS.

  • Эта статья содержит вспомогательную информацию на сайте www.pnas.org/lookup/suppl/doi:10.1073/pnas.1815682116/-/DCSupplemental.

  • Авторские права © 2019 Автор (ы).Опубликовано PNAS.

Матрицы и детерминанты

Автор M Bourne

Зачем изучать Матрицу …?

Матрица — это просто набор чисел, расположенных в прямоугольной таблице.

`((2,4, -1,0), (1,3,7,2))`

Справа — пример матрицы 2 × 4. В нем 2 строки и 4 столбца. Обычно мы пишем матрицы в круглых скобках () или скобках [].

Мы можем складывать, вычитать и умножать матрицы вместе при определенных условиях.

Мы используем матрицы для решения одновременных уравнений, с которыми мы встречались ранее. Матрицы используются для решения задач в:

  • электроника
  • статика
  • робототехника
  • линейное программирование
  • оптимизация
  • пересечений самолетов
  • генетика

В этой главе мы видим некоторые из этих приложений, особенно в Матрицах и линейных уравнениях.

Для больших систем уравнений мы используем компьютер, чтобы найти решение.В этой главе сначала показаны основы матричной арифметики, а затем мы покажем несколько компьютерных примеров (с использованием Scientific Notebook или аналогичных), чтобы вы поняли, что компьютер делает за вас.

Вы можете пропустить следующую часть, если хотите сразу перейти к матрицам.

Детерминанты

Определитель матрицы представляет собой одно число. Мы получаем это значение путем умножения и сложения его элементов особым образом. Мы можем использовать определитель матрицы для решения системы одновременных уравнений.

Например, если у нас есть (квадратная) матрица 2 × 2:

`((5,7), (2, -3))`

, то определитель этой матрицы записывается в вертикальных линиях следующим образом:

`| (5,7), (2, -3) |`

В следующем разделе мы увидим, как оценивать этот детерминант. (Имеет значение -29).

В этой главе

1. Определитель — полученный из квадратной матрицы, определитель необходимо умножить, чтобы получить единственное число.

2. Большие детерминанты — этот раздел поможет вам понять меньшие детерминанты.

3. Матрицы — определение, особенности, единичная матрица и примеры

4. Умножение матриц — как умножать матрицы разного размера. Включает интерактив, где вы можете изучить концепцию.

5. Нахождение обратной матрицы — которую мы используем для решения систем уравнений.

6. Матрицы и линейные уравнения — как решать системы уравнений с матрицами

Мы начинаем главу с введения в детерминанты »

Матрицы

и матричная алгебра — Статистика Как к

Матрицы и содержание матричной алгебры (щелкните, чтобы перейти к этому разделу):

  1. Матричная алгебра: введение
  2. Добавление матрицы: другие примеры
  3. Умножение матриц
  4. Определение сингулярной матрицы
  5. Матрица идентичности
  6. Что такое обратная матрица?
  7. Собственные значения и собственные векторы
  8. Расширенные матрицы
  9. Определитель матрицы
  10. Диагональная матрица
  11. Что такое симметричная и кососимметричная матрицы?
  12. Что такое матрица транспонирования?
  13. Что такое матрица дисперсии-ковариации?
  14. Корреляционные матрицы
  15. Идемпотентная матрица.

Матрица — это прямоугольный массив чисел, упорядоченный по столбцам и строкам (как в электронной таблице). Матричная алгебра используется в статистике для выражения наборов данных. Например, ниже представлен рабочий лист Excel со списком оценок за экзамены:

Преобразование в матричную алгебру в основном просто включает удаление идентификаторов столбцов и строк. Добавляется идентификатор функции (в данном случае «G» для оценок):

Числа, которые появляются в матрице, называются элементами матрицы.

Матрицы

: Обозначение

Почему странная нотация?
Мы используем другую нотацию (в отличие от хранения данных в формате электронной таблицы) по простой причине: соглашение. Соблюдение соглашений упрощает соблюдение правил матричной математики (таких как сложение и вычитание). Например, в элементарной алгебре, если у вас есть список вроде этого: 2 яблока, 3 банана, 5 виноградин, вы должны изменить его на 2a + 3b + 5g, чтобы соблюсти соглашение.

Некоторые из наиболее распространенных терминов, с которыми вы столкнетесь при работе с матрицами:

  • Размер (также называемый порядком): сколько строк и столбцов имеет матрица.Сначала перечислены строки, за ними следуют столбцы. Например, матрица 2 x 3 означает 2 строки и 3 столбца.
  • Элементы : числа, которые появляются внутри матрицы.
  • Матрица идентичности (I): Диагональная матрица с нулями в качестве элементов, за исключением диагонали, в которой есть единицы.
  • Скаляр : любое действительное число.
  • Матрица Функция: скаляр, умноженный на матрицу, чтобы получить другую матрицу.

Матрицы идентичности. Изображение: Википедия.com.

Матричная алгебра: сложение и вычитание

Размер матрицы (т. Е. 2 ​​x 2) также называется размером матрицы , размером или порядком матрицы. Если вы хотите сложить (или вычесть) две матрицы, их размеры должны быть точно так же, как . Другими словами, вы можете добавить матрицу 2 x 2 к другой матрице 2 x 2, но не матрицу 2 x 3. Добавление матриц очень похоже на обычное сложение: вы просто добавляете одинаковые числа в одно и то же место (например, складываете все числа в столбце 1, строке 1 и все числа в столбце 2, строке 2).

Примечание к обозначениям: рабочий лист (например, в Excel) использует буквы столбцов (ABCD) и номера строк (123), чтобы указать местоположение ячейки, например A1 или D2. Для матриц типично использование обозначений типа g ij , что означает i-ю строку и j-й столбец матрицы G.

Матричное вычитание работает точно так же.
В начало

Матричное дополнение — это всего лишь серия дополнений. Для матрицы 2 × 2:

  • Сложите верхние левые числа вместе и запишите сумму в новую матрицу в верхнем левом положении.
  • Сложите верхние правые числа и запишите сумму в верхнем правом углу.
  • Сложите нижние левые числа вместе и запишите сумму в нижнем левом углу.
  • Сложите нижние правые числа вместе и запишите сумму в правом нижнем углу:

Используйте ту же процедуру для матрицы 2 × 3:

Фактически, вы можете использовать этот базовый метод для добавления любых матриц, если ваши матрицы имеют одинаковые размеры (одинаковое количество столбцов и строк).Другими словами, , если матрицы одинакового размера, вы можете их добавить. Если они разного размера, вы не можете их добавить.

  • Матрица с 4 строками и 2 столбцами может быть добавлена ​​ к матрице с 4 строками и 2 столбцами.
  • Матрица с 4 строками и 2 столбцами не может быть добавлена ​​ к матрице с 5 строками и 2 столбцами.

Вышеупомянутый метод иногда называют «начальным суммированием», поскольку вы просто складываете элементы и фиксируете результат.

Другой способ думать об этом…

Подумайте, что представляет собой матрица. Эта очень простая матрица [5 2 5] может представлять 5x + 2y + 5z. И эта матрица [2 1 6] могла бы равняться 2x + y + 6z. Если сложить их вместе с помощью алгебры, получится:
5x + 2y + 5z + 2x + y + 6z = 7x + 3y + 11z.
Это тот же результат, что и при сложении записей в матрицах.

Сложение матрицы для неравных размеров

Если у вас неравные размеры, вы все равно можете сложить матрицы вместе, но вам придется использовать другой (гораздо более продвинутый) метод.Один из таких приемов — прямая сумма. Прямая сумма (⊕) любой пары матриц A размера m × n и B размера p × q представляет собой матрицу размера (m + p) × (n + q):

Например:

В начало

Относительно легко умножить на одно число (так называемое «скалярное умножение»), например 2:

Просто умножьте каждое число в матрице на 2, и вы получите новую матрицу. На изображении выше:
2 * 9 = 18
2 * 3 = 6
2 * 5 = 10
2 * 7 = 14

Результат четырех умножений дает числа в новой матрице справа.

Умножение матриц: две матрицы

Когда вы хотите перемножить две матрицы, процесс становится немного сложнее. Вам нужно умножить строки первой матрицы на столбцы второй матрицы. Другими словами, умножьте по строкам первой матрицы и по столбцам второй матрицы. После того, как вы умножили, сложите продукты и запишите ответы в виде новой матрицы.

Если все это звучит немного сложно, это (очень короткое) видео показывает, как это делается:

Вы можете выполнить матричное умножение для двух матриц, только если количество столбцов в первой матрице равно количеству строк во второй матрице.Например, вы можете умножить матрицу 2 x 3 (две строки и три столбца) на матрицу 3 x 4 (три строки и четыре столбца).

Очевидно, что это может стать очень сложным (и утомительным) процессом. Тем не менее, вы можете найти множество достойных инструментов для умножения матриц в Интернете. Мне нравится этот от Матрицы Решиш. После расчета вы можете умножить результат на другую матрицу и другую, что означает, что вы можете умножить несколько матриц вместе.

Microsoft Excel также может выполнять матричное умножение с использованием функций «массива».Вы можете найти инструкции здесь, на сайте Стэнфорда. Прокрутите вниз до места, где написано «Матричные операции в Excel».
В начало

Быстрый взгляд на матрицу может сказать вам, является ли она сингулярной матрицей. Если матрица квадратная и имеет одну строку или столбец с нулями или , два равных столбца или две равные строки, то это особая матрица. Например, следующие десять матриц являются единственными (изображение: Wolfram):

Существуют и другие типы сингулярных матриц, некоторые не так легко обнаружить.Следовательно, необходимо более формальное определение.

Следующие три свойства определяют сингулярную матрицу:

  1. Матрица квадратная и
  2. Не имеет инверсии.
  3. Имеет определитель 0.

1. Квадратная матрица

Квадратная матрица имеет (как следует из названия) равное количество строк и столбцов. Говоря более формально, вы бы сказали, что матрица из m столбцов и n строк является квадратной, если m = n.Матрицы, которые не являются квадратными, являются прямоугольными.
Сингулярная матрица — это квадратная матрица, но не все квадратные матрицы сингулярны.

Необратимые матрицы

Если квадратная матрица не имеет обратной, то это особая матрица.

Обратная матрица — это то же самое, что и обратная величина числа. Если умножить матрицу на обратную, то получится единичная матрица , , матричный эквивалент 1. Идентификационная матрица в основном представляет собой последовательность единиц и нулей.Идентификационная матрица различается в зависимости от размера матрицы.

Матрицы идентичности. Изображение: Wikipedia.com.

Если вы не знакомы с поиском инверсий, вы можете посмотреть это короткое видео:

Определитель нуля

Определитель — это просто специальное число, которое используется для описания матриц и поиска решений систем линейных уравнений. Формула для вычисления определителя различается в зависимости от размера матрицы.Например, матрица 2 × 2, формула ad-bc.

Эта простая матрица 2 × 2 сингулярна, потому что ее определитель равен нулю:

К началу

Единичная матрица — это квадратная матрица с единицами в качестве элементов на главной диагонали сверху слева направо снизу и нулями в остальных местах. Когда вы умножаете квадратную матрицу на единичную матрицу, исходная квадратная матрица остается неизменной. Например:

По идее аналогичен айдентике. В базовой математике элемент идентичности оставляет число неизменным.Например, кроме того, тождественный элемент равен 0, потому что 1 + 0 = 1, 2 + 0 = 2 и т. Д., А при умножении тождественный элемент равен 1, потому что любое число, умноженное на 1, равно этому числу (т. Е. 10 * 1 = 10 ). В более формальных терминах, если x — действительное число, то число 1 называется мультипликативным тождеством , потому что 1 * x = x и x * 1 = x. По той же логике единичная матрица I получила свое название, потому что для всех матриц A , I * A = A и A * I = A .

В матричной алгебре единичный элемент различается в зависимости от размера матрицы, с которой вы работаете; в отличие от сингулярной единицы для мультипликативной идентичности и 0 для аддитивной идентичности, не существует единой единичной матрицы для всех матриц. Для любой матрицы n * n существует единичная матрица I n * n . На главной диагонали всегда будут единицы, а оставшиеся пробелы — нули. На следующем изображении показаны матрицы идентичности для матрицы 2 x 2 и матрицы 5 x 5:

Дополнительная идентификационная матрица

Когда люди говорят о «матрице идентичности», они обычно имеют в виду мультипликативную матрицу идентичности.Однако есть и другой тип: аддитивная единичная матрица. Когда эта матрица добавляется к другой, вы получаете исходную матрицу. Неудивительно, что каждый элемент в этих матрицах — нули. Поэтому их иногда называют нулевой матрицей .

Аддитивная единичная матрица для матрицы 3 * 3.

Вернуться к началу

Обратные матрицы — это то же самое, что и обратные. В элементарной алгебре (а, возможно, и раньше) вы столкнулись с идеей обратного: одно число, умноженное на другое, может равняться 1.

Изображение предоставлено LTU

Если вы умножите одну матрицу на ее обратную, вы получите матричный эквивалент 1: Identity Matrix , которая по сути представляет собой матрицу с единицами и нулями.

Поиск обратной матрицы состоит из нескольких шагов. Посмотрите это короткое видео о том, как найти обратную матрицу, или выполните следующие действия:

Шаг 1: Найдите адъюгат матрицы. Сопряжение матрицы можно найти, переставив одну диагональ и взяв негативы другой:

Чтобы найти сопряжение матрицы 2 × 2, поменяйте местами диагонали a и d, а затем поменяйте местами знаки c и d.

Шаг 2: Найдите определитель матрицы. Для матрицы
A B C D (см. Изображение выше) определитель равен (a * d) — (b * c).
Шаг 3: Умножить 1 / определитель * адъюгат. .

Проверка ответа

Вы можете проверить свой ответ с помощью умножения матриц. Умножьте свою матрицу ответов на исходную матрицу, и вы получите единичную матрицу. Вы также можете воспользоваться онлайн-калькулятором здесь.
В начало

Собственное значение (λ) — это специальный скаляр, используемый при матричном умножении и имеющий особое значение в нескольких областях физики, включая анализ устойчивости и небольшие колебания колеблющихся систем.Когда вы умножаете матрицу на вектор и получаете тот же вектор в качестве ответа, вместе с новым скаляром, скаляр называется собственным значением . Основное уравнение:
A x = λ x ; мы говорим, что λ является собственным значением A.
Все приведенное выше уравнение говорит о том, что , если вы возьмете матрицу A и умножите ее на вектор x , вы получите то же самое, как если бы вы взяли собственное значение и умножили его по вектору x .

Пример собственного значения

В следующем примере 5 — собственное значение A, а (1,2) — собственный вектор:

Давайте рассмотрим это по шагам, чтобы наглядно продемонстрировать, что такое собственное значение.В обычном умножении, если вы умножаете матрицу n x n на вектор n x 1, в результате вы получаете новый вектор n x 1. На следующем изображении показан этот принцип для матрицы 2 x 2, умноженной на (1,2):

Что если бы вместо новой матрицы nx 1 можно было получить ответ с тем же вектором, который вы умножили на вместе с новым скаляром?

Когда это возможно, вектор умножения (то есть тот, который также есть в ответе) называется собственным вектором, а соответствующий скаляр — собственным значением.Обратите внимание, что я сказал «, когда это возможно» , потому что иногда невозможно вычислить значение для λ. Разложение квадратной матрицы A на собственные значения и собственные векторы (их можно иметь несколько значений для одной и той же матрицы) известно в так называемом разложении по собственным значениям . Разложение на собственные числа всегда возможно, если матрица, состоящая из собственных векторов матрицы A, является квадратной.

Расчет

Найдите собственные значения для следующей матрицы:

Шаг 1: Умножьте единичную матрицу на λ.Единичная матрица для любой матрицы 2 × 2 равна [1 0; 0 1], поэтому:

Шаг 2: Вычтите ответ из шага 1 из матрицы A, используя вычитание матрицы:

Шаг 3: Найдите определитель матрицы, вычисленной на шаге 2:
det = (5- λ) (- 1-λ) — (3) (3)
Упрощая, получаем:
-5 — 5λ + λ + λ 2 — 9
= λ 2 — 4λ — 14

Шаг 4: Установите уравнение, которое вы нашли на шаге 3, равным нулю и решите для λ:
0 = λ 2 — 4λ — 14 = 2
Мне нравится использовать свой TI-83 для поиска корней, но вы также можете использовать алгебру или это онлайн-калькулятор.Находя корни (нули), получаем x = 2 + 3√2, 2 — 3√2

Ответ : 2 + 3√2 и 2-3√2

Математика для больших матриц такая же, но вычисления могут быть очень сложными. Для матриц 3 × 3 используйте калькулятор внизу этого раздела; для больших матриц попробуйте этот онлайн-калькулятор.

В начало

На изображении выше показана расширенная матрица (A | B) внизу. Расширенные матрицы обычно используются для решения систем линейных уравнений и, собственно, именно поэтому они были впервые разработаны.Три столбца слева от полосы представляют коэффициенты (по одному столбцу для каждой переменной). Эта область называется матрицей коэффициентов . Последний столбец справа от полосы представляет собой набор констант (т. Е. Значений справа от знака равенства в наборе уравнений). Она называется расширенной матрицей , потому что матрица коэффициентов была «дополнена» значениями после знака равенства.

Например, следующая система линейных уравнений:

x + 2y + 3z = 0
3x + 4y + 7z = 2
6x + 5y + 9z = 11

Можно поместить в следующую расширенную матрицу:

После того, как вы поместили свою систему в расширенную матрицу, вы можете выполнять операции со строками для решения системы.

У вас не , а , чтобы использовать вертикальную полосу в расширенной матрице. Обычно матрицы вообще не содержат линий. Полоса просто упрощает отслеживание ваших коэффициентов и ваших констант справа от знака равенства. Если вы вообще используете вертикальную полосу, зависит от учебника, который вы используете, и от предпочтений вашего преподавателя.

Написание системы уравнений

Вы также можете работать в обратном направлении, чтобы написать систему линейных уравнений, заданную расширенной матрицей.
Пример вопроса: Напишите систему линейных уравнений для следующей матрицы.

Шаг 1: Запишите коэффициенты для первого столбца, за которым следует «x». Обязательно укажите положительные или отрицательные числа:
-1x
2x
6x
Шаг 2: Напишите коэффициенты для второго столбца, а затем укажите «y». Сложите, если это положительное число, вычтите, если оно отрицательное:
-1x + 7y
2x + 4y
6x + 2y
Шаг 3: Напишите коэффициенты для второго столбца, после чего укажите «z.«Сложите, если это положительное число, и вычтите, если оно отрицательное:
-1x + 7y + 3
2x + 4y — 7
6x + 2y + 9
Шаг 3. Запишите константы в третьем столбце со знаком равенства.
-1x + 7y + 3 = 0
2x + 4y — 7 = 2
6x + 2y + 9 = 7
Примечание : если на этом шаге стоит отрицательный знак, просто сделайте константу отрицательным числом.
В начало

Определитель матрицы — это просто специальное число, которое используется для описания матриц для поиска решений систем линейных уравнений, нахождения обратных матриц и для различных приложений в исчислении.Определить на простом английском языке невозможно; обычно его определяют в математических терминах или в терминах того, что он может вам помочь. Определитель матрицы имеет несколько свойств:

  • Это реальный номер. Сюда входят отрицательные числа.
  • Определители существуют только для квадратных матриц.
  • Обратная матрица существует только для матриц с ненулевыми определителями.

Символ для определителя матрицы A — | A |, который также является тем же символом, который используется для абсолютного значения, хотя эти два символа не имеют ничего общего друг с другом.

Формула для вычисления определителя матрицы различается в зависимости от размера матрицы.

Определитель матрицы 2 × 2

Формула определителя матрицы 2 × 2 — ad-bc. Другими словами, умножьте верхний левый элемент на нижний правый, затем вычтите произведение верхнего правого и нижнего левого.

Определитель матрицы 3 × 3

Определитель матрицы 3 × 3 находится по следующей формуле:
| A | = a (ei — fh) — b (di — fg) + c (dh — eg)
Это может показаться сложным, но если вы пометили элементы с помощью a, b, c в верхнем ряду, d, e, f во второй строке и g, h, i в последней, становится основной арифметикой.
Пример :
Найдите определитель следующей матрицы 3 × 3:

= 3 (6 × 2-7 × 3) –5 (2 × 2-7 × 4) +4 (2 × 3-6 × 4)
= -219
По сути, здесь происходит умножение a, b и d на детерминанты меньших 2×2 в матрице 3×3. Этот шаблон продолжается для поиска определителей матриц более высокого порядка.

Определитель матрицы 4 × 4

Чтобы найти определитель матрицы 4 × 4, вам сначала нужно найти определители четырех матриц 3 × 3, которые находятся в матрице 4 × 4.В виде формулы:

Вернуться к началу

Диагональная матрица — это симметричная матрица со всеми нулями, кроме ведущей диагонали, которая проходит от верхнего левого угла до нижнего правого.

Записи на самой диагонали также могут быть нулями; любую квадратную матрицу со всеми нулями еще можно назвать диагональной матрицей.

Единичная матрица, которая имеет все 1s по диагонали, также является диагональной матрицей. Любая матрица с равными элементами по диагонали (т. Е.2,2,2 или 9,9,9), является скалярным кратным единичной матрицы и также может быть классифицировано как диагональное.

Диагональная матрица имеет максимум n чисел, которые не равны нулю, где n — порядок матрицы. Например, матрица 3 x 3 (порядок 3) имеет диагональ, состоящую из 3 чисел, а матрица 5 x 5 (порядок 5) имеет диагональ из 5 чисел.

Обозначение

Обозначение, обычно используемое для описания диагональной матрицы: diag (a, b, c) , где abc представляет собой числа в первой диагонали.Для приведенной выше матрицы это обозначение будет diag (3,2,4). .

Верхняя и нижняя треугольные матрицы

Диагональ матрицы всегда относится к ведущей диагонали. Ведущая диагональ в матрице помогает определить два других типа матриц: нижнетреугольные матрицы и верхние треугольные матрицы. В нижнетреугольной матрице числа под диагональю; верхнетреугольная матрица имеет числа над диагональю.

Диагональная матрица — это матрица с нижней диагональю и матрица с нижней диагональю.

Прямоугольные диагональные матрицы

Для наиболее распространенного использования диагональная матрица представляет собой квадратную матрицу с порядком (размером) n . Существуют и другие формы, которые обычно не используются, например прямоугольная диагональная матрица . Матрица этого типа также имеет одну ведущую диагональ с числами, а остальные элементы — нули. Ведущая диагональ берется из наибольшего квадрата неквадратной матрицы.

В начало

Транспонирование матрицы (или транспонирование матрицы) — это как раз то место, где вы переключаете все строки матрицы в столбцы.Матрицы транспонирования полезны при комплексном умножении.

Альтернативный способ описания транспонированной матрицы состоит в том, что элемент в строке «r» и столбце «c» транспонируется в строку «c» и столбец «r». Например, элемент в строке 2, столбце 3 будет транспонирован в столбец 2, строку 3. Размер матрицы также изменится. Например, если у вас есть матрица 4 x 5, вы бы транспонировали ее в матрицу 5 x 4.

Симметричная матрица — это частный случай транспонированной матрицы; он равен своей транспонированной матрице.

Говоря более формально, A = A T .

Символы для матрицы транспонирования

Обычный символ для транспонированной матрицы — A T Однако Wolfram Mathworld утверждает, что также используются два других символа: A и.

Свойства матриц транспонирования

Свойства транспонированных матриц аналогичны основным числовым свойствам, с которыми вы столкнулись в базовой алгебре (например, ассоциативным и коммутативным). Основные свойства матриц:

  • (A T ) T = A: транспонированная матрица транспонирования является исходной матрицей.
  • (A + B) T = A T + B T : Транспонирование двух сложенных вместе матриц такое же, как транспонирование каждой отдельной матрицы, сложенной вместе.
  • (rA) T = rA T : когда матрица умножается на скалярный элемент, не имеет значения, в каком порядке вы транспонируете (примечание: скалярный элемент — это величина, которая может умножать матрицу).
  • (AB) T = B T A T : транспонирование двух матриц, умноженных вместе, совпадает с произведением их матриц транспонирования в обратном порядке.
  • (A -1 ) T = (A T ) -1 : транспонирование и инверсия матрицы могут выполняться в любом порядке.

Вернуться к началу

Симметричная матрица — это квадратная матрица, имеющая симметрию относительно ведущей диагонали, сверху слева направо. Представьте себе складку в матрице по диагонали (не включайте числа в действительную диагональ). Верхняя правая половина матрицы и нижняя левая половина являются зеркальными отображениями относительно диагонали:

Если вы можете сопоставить числа друг с другом по линии симметрии ( всегда ведущая диагональ), как в примере справа , у вас симметричная матрица.

Альтернативное определение

Другой способ определить симметричную матрицу состоит в том, что симметричная матрица равна ее транспонированной. A транспонирование матрицы — это когда первая строка становится первым столбцом, вторая строка становится вторым столбцом, третья строка становится третьим столбцом… и так далее. Вы просто превращаете строки в столбцы.

Если вы возьмете симметричную матрицу и транспонируете ее, матрица будет выглядеть точно так же, отсюда и альтернативное определение, что симметричная матрица равна ее транспонированию.С математической точки зрения, M = M T , где M T — матрица транспонирования.

Максимальное количество номеров

Поскольку большинство чисел в симметричной матрице дублируются, существует ограничение на количество различных чисел, которые она может содержать. Уравнение для максимального количества чисел в матрице порядка n: n (n + 1) / 2. Например, в симметричной матрице 4-го порядка, подобной приведенной выше, имеется максимум 4 (4 + 1) / 2 = 10 различных чисел. Это имеет смысл, если подумать: диагональ — это четыре числа, и если вы сложите числа в нижней левой половине (исключая диагональ), вы получите 6.

Диагональные матрицы

Диагональная матрица — это частный случай симметричной матрицы. Диагональная матрица имеет все нули, кроме ведущей диагонали.

Что такое асимметричная матрица?

Кососимметричная матрица, иногда называемая антисимметричной матрицей , представляет собой квадратную матрицу, симметричную относительно обеих диагоналей. Например, следующая матрица является асимметричной:

Математически асимметричная матрица удовлетворяет условию a ij = -a ji .Например, возьмите запись в строке 3, столбец 2, которая равна 4. Его симметричным аналогом является -4 в строке 2, столбце 3. Это условие также можно записать в терминах его транспонированной матрицы: A T = — А. Другими словами, матрица является кососимметричной, только если A T = -A, где A T — это транспонированная матрица.

Все старшие диагональные элементы в кососимметричной матрице должны быть нулевыми. Это потому, что i, i = −a i, i влечет i, i = 0.

Еще одним интересным свойством этого типа матрицы является то, что если у вас есть две кососимметричные матрицы A и B одинакового размера, вы также получите кососимметричную матрицу, если сложите их вместе:

Добавление двух кососимметричных матриц вместе.

Этот факт может помочь вам доказать, что две матрицы кососимметричны. Первый шаг — убедиться, что все элементы на ведущей диагонали равны нулю (что невозможно «доказать» математически!).Второй шаг — сложение матриц. Если результатом является третья матрица, которая является кососимметричной, то вы доказали, что a ij = — a ji .

Косоэрмитский

Косоэрмитова матрица по сути такая же, как кососимметричная матрица, за исключением того, что косоэрмитова матрица может содержать комплексные числа.

Косоэрмитова матрица, показывающая комплексные числа.

Фактически, кососимметричный и косоэрмитовый эквивалентны для вещественных матриц (матрицы, которая почти полностью состоит из действительных чисел).
Старшая диагональ косоэрмитовой матрицы должна содержать чисто мнимые числа; в мнимой сфере ноль считается мнимым числом.
Вернуться к началу

Матрица ковариации и дисперсии (также называемая матрицей ковариации или матрицей дисперсии) — это квадратная матрица, которая отображает дисперсию и ковариацию двух наборов двумерных данных вместе. Разница — это мера того, насколько разбросаны данные. Ковариация — это мера того, насколько две случайные величины перемещаются вместе в одном направлении.

Дисперсии отображаются в диагональных элементах, а ковариации между парами переменных отображаются в недиагональных элементах. Дисперсии находятся в диагоналях ковариантной матрицы, потому что в основном эти дисперсии являются ковариатами каждой отдельной переменной с самой собой.

Следующая матрица показывает дисперсию для A (2,00), B (3,20) и C (0,21) в диагональных элементах.

Ковариации для каждой пары показаны в других ячейках.Например, ковариация для A и B равна -0,21, а ковариация для A и C равна -0,10. Вы можете смотреть в столбец и строку или строку и столбец (например, AC или CA), чтобы получить тот же результат, потому что ковариация для A и C такая же, как ковариация для C и A. Следовательно, ковариация дисперсии матрица также является симметричной матрицей.

Создание матрицы дисперсии-ковариации

Многие статистические пакеты, включая Microsoft Excel и SPSS, могут создавать ковариативно-вариативные матрицы. Обратите внимание, что Excel вычисляет ковариацию для генеральной совокупности (знаменатель n), а не для выборки (n-1).Это может привести к немного неправильным вычислениям для матрицы дисперсии-ковариации. Чтобы исправить это, вам нужно умножить каждую ячейку на n / n-1.

Если вы хотите сделать один вручную:
Шаг 1: Вставьте отклонения для ваших данных в диагонали матрицы.
Шаг 2: Рассчитайте ковариацию для каждой пары и введите их в соответствующую ячейку. Например, ковариация для A / B в приведенном выше примере появляется в двух местах (A B и B A).

Добавить комментарий

Ваш адрес email не будет опубликован. Обязательные поля помечены *